Programs & Examples On #System.net.webexception

system.net.webexception is a .NET exception is thrown when an error occurs while accessing the network through a pluggable protocol.

System.Net.WebException: The remote name could not be resolved:

I had a similar issue when trying to access a service (old ASMX service). The call would work when accessing via an IP however when calling with an alias I would get the remote name could not be resolved.

Added the following to the config and it resolved the issue:

<system.net>
    <defaultProxy enabled="true">
    </defaultProxy>
</system.net>

No connection could be made because the target machine actively refused it?

One more possibility --

Make sure you're trying to open the same IP address as where you're listening. My server app was listening to the host machine's IP address using IPv6, but the client was attempting to connect on the host machine's IPv4 address.

Hibernate openSession() vs getCurrentSession()

SessionFactory: "One SessionFactory per application per DataBase" ( e.g., if you use 3 DataBase's in our application , you need to create sessionFactory object per each DB , totally you need to create 3 sessionFactorys . or else if you have only one DataBase One sessionfactory is enough ).

Session: "One session for one request-response cycle". you can open session when request came and you can close session after completion of request process. Note:-Don't use one session for web application.

What are the differences between Visual Studio Code and Visual Studio?

One huge difference (for me) is that Visual Studio Code is one monitor only. With Visual Studio you can use multi-screen setups.

newline in <td title="">

Using &#013; didn't work in my fb app. However this did, beautifully (in Chrome FF and IE):

<img src="'../images/foo.gif'" title="line 1&lt;br&gt;line 2">

Changing WPF title bar background color

Here's an example on how to achieve this:

    <Grid DockPanel.Dock="Right"
      HorizontalAlignment="Right">

        <StackPanel Orientation="Horizontal"
                HorizontalAlignment="Right"
                VerticalAlignment="Center">

            <Button x:Name="MinimizeButton"
                KeyboardNavigation.IsTabStop="False"
                Click="MinimizeWindow"
                Style="{StaticResource MinimizeButton}" 
                Template="{StaticResource MinimizeButtonControlTemplate}" />

            <Button x:Name="MaximizeButton"
                KeyboardNavigation.IsTabStop="False"
                Click="MaximizeClick"
                Style="{DynamicResource MaximizeButton}" 
                Template="{DynamicResource MaximizeButtonControlTemplate}" />

            <Button x:Name="CloseButton"
                KeyboardNavigation.IsTabStop="False"
                Command="{Binding ApplicationCommands.Close}"
                Style="{DynamicResource CloseButton}" 
                Template="{DynamicResource CloseButtonControlTemplate}"/>

        </StackPanel>
    </Grid>
</DockPanel>

Handle Click Events in the code-behind.

For MouseDown -

App.Current.MainWindow.DragMove();

For Minimize Button -

App.Current.MainWindow.WindowState = WindowState.Minimized;

For DoubleClick and MaximizeClick

if (App.Current.MainWindow.WindowState == WindowState.Maximized)
{
    App.Current.MainWindow.WindowState = WindowState.Normal;
}
else if (App.Current.MainWindow.WindowState == WindowState.Normal)
{
    App.Current.MainWindow.WindowState = WindowState.Maximized;
}

How to Run a jQuery or JavaScript Before Page Start to Load

This should do the trick:

window.onload = function(event) {
    event.stopPropagation(true);
    window.location.href="http://www.google.com";
};

Good luck  ;)

Retrieve only the queried element in an object array in MongoDB collection

The syntax for find in mongodb is

    db.<collection name>.find(query, projection);

and the second query that you have written, that is

    db.test.find(
    {shapes: {"$elemMatch": {color: "red"}}}, 
    {"shapes.color":1})

in this you have used the $elemMatch operator in query part, whereas if you use this operator in the projection part then you will get the desired result. You can write down your query as

     db.users.find(
     {"shapes.color":"red"},
     {_id:0, shapes: {$elemMatch : {color: "red"}}})

This will give you the desired result.

ArrayList initialization equivalent to array initialization

This is how it is done using the fluent interface of the op4j Java library (1.1. was released Dec '10) :-

List<String> names = Op.onListFor("Ryan", "Julie", "Bob").get();

It's a very cool library that saves you a tonne of time.

Multiple cases in switch statement

.NET Framework 3.5 has got ranges:

Enumerable.Range from MSDN

you can use it with "contains" and the IF statement, since like someone said the SWITCH statement uses the "==" operator.

Here an example:

int c = 2;
if(Enumerable.Range(0,10).Contains(c))
    DoThing();
else if(Enumerable.Range(11,20).Contains(c))
    DoAnotherThing();

But I think we can have more fun: since you won't need the return values and this action doesn't take parameters, you can easily use actions!

public static void MySwitchWithEnumerable(int switchcase, int startNumber, int endNumber, Action action)
{
    if(Enumerable.Range(startNumber, endNumber).Contains(switchcase))
        action();
}

The old example with this new method:

MySwitchWithEnumerable(c, 0, 10, DoThing);
MySwitchWithEnumerable(c, 10, 20, DoAnotherThing);

Since you are passing actions, not values, you should omit the parenthesis, it's very important. If you need function with arguments, just change the type of Action to Action<ParameterType>. If you need return values, use Func<ParameterType, ReturnType>.

In C# 3.0 there is no easy Partial Application to encapsulate the fact the the case parameter is the same, but you create a little helper method (a bit verbose, tho).

public static void MySwitchWithEnumerable(int startNumber, int endNumber, Action action){ 
    MySwitchWithEnumerable(3, startNumber, endNumber, action); 
}

Here an example of how new functional imported statement are IMHO more powerful and elegant than the old imperative one.

How to change the DataTable Column Name?

Rename the Column by doing the following:

dataTable.Columns["ColumnName"].ColumnName = "newColumnName";

Main differences between SOAP and RESTful web services in Java

REST is an architecture.
REST will give human-readable results.
REST is stateless.
REST services are easily cacheable.

SOAP is a protocol. It can run on top of JMS, FTP, and HTTP.

jQuery get the rendered height of an element?

document.querySelector('.project_list_div').offsetHeight;

OSError: [Errno 2] No such file or directory while using python subprocess in Django

No such file or directory can be also raised if you are trying to put a file argument to Popen with double-quotes.

For example:

call_args = ['mv', '"path/to/file with spaces.txt"', 'somewhere']

In this case, you need to remove double-quotes.

call_args = ['mv', 'path/to/file with spaces.txt', 'somewhere']

Jackson: how to prevent field serialization

Aside from @JsonIgnore, there are a couple of other possibilities:

  • Use JSON Views to filter out fields conditionally (by default, not used for deserialization; in 2.0 will be available but you can use different view on serialization, deserialization)
  • @JsonIgnoreProperties on class may be useful

ActionBarActivity cannot resolve a symbol

If the same error occurs in ADT/Eclipse

Add Action Bar Sherlock library in your project.

Now, to remove the "import The import android.support.v7 cannot be resolved" error download a jar file named as android-support-v7-appcompat.jar and add it in your project lib folder.

This will surely removes your both errors.

How to test that no exception is thrown?

You're approaching this the wrong way. Just test your functionality: if an exception is thrown the test will automatically fail. If no exception is thrown, your tests will all turn up green.

I have noticed this question garners interest from time to time so I'll expand a little.

Background to unit testing

When you're unit testing it's important to define to yourself what you consider a unit of work. Basically: an extraction of your codebase that may or may not include multiple methods or classes that represents a single piece of functionality.

Or, as defined in The art of Unit Testing, 2nd Edition by Roy Osherove, page 11:

A unit test is an automated piece of code that invokes the unit of work being tested, and then checks some assumptions about a single end result of that unit. A unit test is almost always written using a unit testing framework. It can be written easily and runs quickly. It's trustworthy, readable, and maintainable. It's consistent in its results as long as production code hasn't changed.

What is important to realize is that one unit of work usually isn't just one method but at the very basic level it is one method and after that it is encapsulated by other unit of works.

enter image description here

Ideally you should have a test method for each separate unit of work so you can always immediately view where things are going wrong. In this example there is a basic method called getUserById() which will return a user and there is a total of 3 unit of works.

The first unit of work should test whether or not a valid user is being returned in the case of valid and invalid input.
Any exceptions that are being thrown by the datasource have to be handled here: if no user is present there should be a test that demonstrates that an exception is thrown when the user can't be found. A sample of this could be the IllegalArgumentException which is caught with the @Test(expected = IllegalArgumentException.class) annotation.

Once you have handled all your usecases for this basic unit of work, you move up a level. Here you do exactly the same, but you only handle the exceptions that come from the level right below the current one. This keeps your testing code well structured and allows you to quickly run through the architecture to find where things go wrong, instead of having to hop all over the place.

Handling a tests' valid and faulty input

At this point it should be clear how we're going to handle these exceptions. There are 2 types of input: valid input and faulty input (the input is valid in the strict sense, but it's not correct).

When you work with valid input you're setting the implicit expectancy that whatever test you write, will work.

Such a method call can look like this: existingUserById_ShouldReturn_UserObject. If this method fails (e.g.: an exception is thrown) then you know something went wrong and you can start digging.

By adding another test (nonExistingUserById_ShouldThrow_IllegalArgumentException) that uses the faulty input and expects an exception you can see whether your method does what it is supposed to do with wrong input.

TL;DR

You were trying to do two things in your test: check for valid and faulty input. By splitting this into two method that each do one thing, you will have much clearer tests and a much better overview of where things go wrong.

By keeping the layered unit of works in mind you can also reduce the amount of tests you need for a layer that is higher in the hierarchy because you don't have to account for every thing that might have gone wrong in the lower layers: the layers below the current one are a virtual guarantee that your dependencies work and if something goes wrong, it's in your current layer (assuming the lower layers don't throw any errors themselves).

How to compile makefile using MinGW?

I found a very good example here: https://bigcode.wordpress.com/2016/12/20/compiling-a-very-basic-mingw-windows-hello-world-executable-in-c-with-a-makefile/

It is a simple Hello.c (you can use c++ with g++ instead of gcc) using the MinGW on windows.

The Makefile looking like:

EXECUTABLE = src/Main.cpp

CC = "C:\MinGW\bin\g++.exe"
LDFLAGS = -lgdi32

src = $(wildcard *.cpp)
obj = $(src:.cpp=.o)

all: myprog

myprog: $(obj)
    $(CC) -o $(EXECUTABLE) $^ $(LDFLAGS)

.PHONY: clean
clean:
    del $(obj) $(EXECUTABLE)

How do I set a JLabel's background color?

The JLabel background is transparent by default. Set the opacity at true like that:

label.setOpaque(true);

How to disable Django's CSRF validation?

For Django 2:

from django.utils.deprecation import MiddlewareMixin


class DisableCSRF(MiddlewareMixin):
    def process_request(self, request):
        setattr(request, '_dont_enforce_csrf_checks', True)

That middleware must be added to settings.MIDDLEWARE when appropriate (in your test settings for example).

Note: the setting isn't not called MIDDLEWARE_CLASSES anymore.

Java project in Eclipse: The type java.lang.Object cannot be resolved. It is indirectly referenced from required .class files

Here is how I solved it: In Java-ADT: Windows - Preference - Java - Installed JREs Just add another JRE, pointing to the 'jre' folder under your JDK folder. (jre is included in the jdk). Make sure you chose the new jre.

How do I use a file grep comparison inside a bash if/else statement?

if takes a command and checks its return value. [ is just a command.

if grep -q ...
then
  ....
else
  ....
fi

Remove a prefix from a string

I think you can use methods of the str type to do this. There's no need for regular expressions:

def remove_prefix(text, prefix):
    if text.startswith(prefix): # only modify the text if it starts with the prefix
         text = text.replace(prefix, "", 1) # remove one instance of prefix
    return text

Proper way to exit command line program?

if you do ctrl-z and then type exit it will close background applications.

Ctrl+Q is another good way to kill the application.

How to run a command in the background and get no output?

Sorry this is a bit late but found the ideal solution for somple commands where you don't want any standard or error output (credit where it's due: http://felixmilea.com/2014/12/running-bash-commands-background-properly/)

This redirects output to null and keeps screen clear:

command &>/dev/null &

Importing text file into excel sheet

There are many ways you can import Text file to the current sheet. Here are three (including the method that you are using above)

  1. Using a QueryTable
  2. Open the text file in memory and then write to the current sheet and finally applying Text To Columns if required.
  3. If you want to use the method that you are currently using then after you open the text file in a new workbook, simply copy it over to the current sheet using Cells.Copy

Using a QueryTable

Here is a simple macro that I recorded. Please amend it to suit your needs.

Sub Sample()
    With ActiveSheet.QueryTables.Add(Connection:= _
        "TEXT;C:\Sample.txt", Destination:=Range("$A$1") _
        )
        .Name = "Sample"
        .FieldNames = True
        .RowNumbers = False
        .FillAdjacentFormulas = False
        .PreserveFormatting = True
        .RefreshOnFileOpen = False
        .RefreshStyle = xlInsertDeleteCells
        .SavePassword = False
        .SaveData = True
        .AdjustColumnWidth = True
        .RefreshPeriod = 0
        .TextFilePromptOnRefresh = False
        .TextFilePlatform = 437
        .TextFileStartRow = 1
        .TextFileParseType = xlDelimited
        .TextFileTextQualifier = xlTextQualifierDoubleQuote
        .TextFileConsecutiveDelimiter = False
        .TextFileTabDelimiter = True
        .TextFileSemicolonDelimiter = False
        .TextFileCommaDelimiter = True
        .TextFileSpaceDelimiter = False
        .TextFileColumnDataTypes = Array(1, 1, 1, 1, 1, 1)
        .TextFileTrailingMinusNumbers = True
        .Refresh BackgroundQuery:=False
    End With
End Sub

Open the text file in memory

Sub Sample()
    Dim MyData As String, strData() As String

    Open "C:\Sample.txt" For Binary As #1
    MyData = Space$(LOF(1))
    Get #1, , MyData
    Close #1
    strData() = Split(MyData, vbCrLf)
End Sub

Once you have the data in the array you can export it to the current sheet.

Using the method that you are already using

Sub Sample()
    Dim wbI As Workbook, wbO As Workbook
    Dim wsI As Worksheet

    Set wbI = ThisWorkbook
    Set wsI = wbI.Sheets("Sheet1") '<~~ Sheet where you want to import

    Set wbO = Workbooks.Open("C:\Sample.txt")

    wbO.Sheets(1).Cells.Copy wsI.Cells

    wbO.Close SaveChanges:=False
End Sub

FOLLOWUP

You can use the Application.GetOpenFilename to choose the relevant file. For example...

Sub Sample()
    Dim Ret

    Ret = Application.GetOpenFilename("Prn Files (*.prn), *.prn")

    If Ret <> False Then
        With ActiveSheet.QueryTables.Add(Connection:= _
        "TEXT;" & Ret, Destination:=Range("$A$1"))

            '~~> Rest of the code

        End With
    End If
End Sub

How to do if-else in Thymeleaf?

Another solution - you can use local variable:

<div th:with="expr_result = ${potentially_complex_expression}">
    <div th:if="${expr_result}">
        <h2>Hello!</h2>
    </div>
    <div th:unless="${expr_result}">
        <span class="xxx">Something else</span>
    </div>
</div>

More about local variables:
http://www.thymeleaf.org/doc/tutorials/2.1/usingthymeleaf.html#local-variables

Sorting an array of objects by property values

For sorting a array you must define a comparator function. This function always be different on your desired sorting pattern or order(i.e. ascending or descending).

Let create some functions that sort an array ascending or descending and that contains object or string or numeric values.

function sorterAscending(a,b) {
    return a-b;
}

function sorterDescending(a,b) {
    return b-a;
}

function sorterPriceAsc(a,b) {
    return parseInt(a['price']) - parseInt(b['price']);
}

function sorterPriceDes(a,b) {
    return parseInt(b['price']) - parseInt(b['price']);
}

Sort numbers (alphabetically and ascending):

var fruits = ["Banana", "Orange", "Apple", "Mango"];
fruits.sort();

Sort numbers (alphabetically and descending):

var fruits = ["Banana", "Orange", "Apple", "Mango"];
fruits.sort();
fruits.reverse();

Sort numbers (numerically and ascending):

var points = [40,100,1,5,25,10];
points.sort(sorterAscending());

Sort numbers (numerically and descending):

var points = [40,100,1,5,25,10];
points.sort(sorterDescending());

As above use sorterPriceAsc and sorterPriceDes method with your array with desired key.

homes.sort(sorterPriceAsc()) or homes.sort(sorterPriceDes())

Complex nesting of partials and templates

UPDATE: Check out AngularUI's new project to address this problem


For subsections it's as easy as leveraging strings in ng-include:

<ul id="subNav">
  <li><a ng-click="subPage='section1/subpage1.htm'">Sub Page 1</a></li>
  <li><a ng-click="subPage='section1/subpage2.htm'">Sub Page 2</a></li>
  <li><a ng-click="subPage='section1/subpage3.htm'">Sub Page 3</a></li>
</ul>
<ng-include src="subPage"></ng-include>

Or you can create an object in case you have links to sub pages all over the place:

$scope.pages = { page1: 'section1/subpage1.htm', ... };
<ul id="subNav">
  <li><a ng-click="subPage='page1'">Sub Page 1</a></li>
  <li><a ng-click="subPage='page2'">Sub Page 2</a></li>
  <li><a ng-click="subPage='page3'">Sub Page 3</a></li>
</ul>
<ng-include src="pages[subPage]"></ng-include>

Or you can even use $routeParams

$routeProvider.when('/home', ...);
$routeProvider.when('/home/:tab', ...);
$scope.params = $routeParams;
<ul id="subNav">
  <li><a href="#/home/tab1">Sub Page 1</a></li>
  <li><a href="#/home/tab2">Sub Page 2</a></li>
  <li><a href="#/home/tab3">Sub Page 3</a></li>
</ul>
<ng-include src=" '/home/' + tab + '.html' "></ng-include>

You can also put an ng-controller at the top-most level of each partial

Convert object array to hash map, indexed by an attribute value of the Object

Using simple Javascript

var createMapFromList = function(objectList, property) {
    var objMap = {};
    objectList.forEach(function(obj) {
      objMap[obj[property]] = obj;
    });
    return objMap;
  };
// objectList - the array  ;  property - property as the key

how to make a new line in a jupyter markdown cell

The double space generally works well. However, sometimes the lacking newline in the PDF still occurs to me when using four pound sign sub titles #### in Jupyter Notebook, as the next paragraph is put into the subtitle as a single paragraph. No amount of double spaces and returns fixed this, until I created a notebook copy 'v. PDF' and started using a single backslash '\' which also indents the next paragraph nicely:

#### 1.1 My Subtitle  \

1.1 My Subtitle
    Next paragraph text.

An alternative to this, is to upgrade the level of your four # titles to three # titles, etc. up the title chain, which will remove the next paragraph indent and format the indent of the title itself (#### My Subtitle ---> ### My Subtitle).

### My Subtitle


1.1 My Subtitle

Next paragraph text.

Disable ScrollView Programmatically?

You can extend the gallery and use some flag to disable scrolling when you want:

public class MyGallery extends Gallery {

public boolean canScroll;

public MyGallery(Context context, AttributeSet attrs) {
    canScroll = true;
    super(context, attrs);
}

public void setCanScroll(boolean flag)
{
    canScroll = flag;
}

@Override
public boolean onScroll(android.view.MotionEvent e1, android.view.MotionEvent e2, float distanceX, float distanceY) {
    if (canScroll)
        return super.onScroll(e1,e2,distancex,distancey);
    else
        return false;
}

@Override
public boolean onSingleTapUp(MotionEvent e)
{
    if (canScroll)
        return super.onSingleTapUp(ey);
    else
        return false;
}

@Override
public boolean onFling(MotionEvent e1, MotionEvent e2, float velocityX, float velocityY)
{
    if (canScroll)
        return super.onFling(e1,e2,velocityX,velocityY);
    else
        return false;
}
}

Convert string into Date type on Python

While it seems the question was answered per the OP's request, none of the answers give a good way to get a datetime.date object instead of a datetime.datetime. So for those searching and finding this thread:

datetime.date has no .strptime method; use the one on datetime.datetime instead and then call .date() on it to receive the datetime.date object.

Like so:

>>> from datetime import datetime
>>> datetime.strptime('2014-12-04', '%Y-%m-%d').date()
datetime.date(2014, 12, 4)

What is Domain Driven Design?

I believe the following pdf will give you the bigger picture. Domain Driven Design by Eric Evans

NOTE: Think of a project you can work on, apply the little things you understood and see best practices. It will help you to grow your ability to the micro service architecture design approach too.

Accessing dict_keys element by index in Python3

Try this

keys = [next(iter(x.keys())) for x in test]
print(list(keys))

The result looks like this. ['foo', 'hello']

You can find more possible solutions here.

iOS 7 - Failing to instantiate default view controller

This warning is also reported if you have some code like:

    window = UIWindow(frame: UIScreen.mainScreen().bounds)
    window?.rootViewController = myAwesomeRootViewController
    window?.makeKeyAndVisible()

In this case, go to the first page of target settings and set Main Interface to empty, since you don't need a storyboard entry for your app.

add a string prefix to each value in a string column using Pandas

If you load you table file with dtype=str
or convert column type to string df['a'] = df['a'].astype(str)
then you can use such approach:

df['a']= 'col' + df['a'].str[:]

This approach allows prepend, append, and subset string of df.
Works on Pandas v0.23.4, v0.24.1. Don't know about earlier versions.

How to find an object in an ArrayList by property

For finding objects which are meaningfully equal, you need to override equals and hashcode methods for the class. You can find a good tutorial here.

http://www.thejavageek.com/2013/06/28/significance-of-equals-and-hashcode/

Java - How to create a custom dialog box?

If you don't need much in the way of custom behavior, JOptionPane is a good time saver. It takes care of the placement and localization of OK / Cancel options, and is a quick-and-dirty way to show a custom dialog without needing to define your own classes. Most of the time the "message" parameter in JOptionPane is a String, but you can pass in a JComponent or array of JComponents as well.

Example:

JTextField firstName = new JTextField();
JTextField lastName = new JTextField();
JPasswordField password = new JPasswordField();
final JComponent[] inputs = new JComponent[] {
        new JLabel("First"),
        firstName,
        new JLabel("Last"),
        lastName,
        new JLabel("Password"),
        password
};
int result = JOptionPane.showConfirmDialog(null, inputs, "My custom dialog", JOptionPane.PLAIN_MESSAGE);
if (result == JOptionPane.OK_OPTION) {
    System.out.println("You entered " +
            firstName.getText() + ", " +
            lastName.getText() + ", " +
            password.getText());
} else {
    System.out.println("User canceled / closed the dialog, result = " + result);
}

SQL error "ORA-01722: invalid number"

Well it also can be :

SELECT t.col1, t.col2, ('test' + t.col3) as test_col3 
FROM table t;

where for concatenation in oracle is used the operator || not +.

In this case you get : ORA-01722: invalid number ...

Bulk Record Update with SQL

You can do this through a regular UPDATE with a JOIN

UPDATE T1
SET Description = T2.Description
   FROM Table1 T1
      JOIN Table2 T2
         ON T2.ID = T1.DescriptionId

In R, dealing with Error: ggplot2 doesn't know how to deal with data of class numeric

The error happens because of you are trying to map a numeric vector to data in geom_errorbar: GVW[1:64,3]. ggplot only works with data.frame.

In general, you shouldn't subset inside ggplot calls. You are doing so because your standard errors are stored in four separate objects. Add them to your original data.frame and you will be able to plot everything in one call.

Here with a dplyr solution to summarise the data and compute the standard error beforehand.

library(dplyr)
d <- GVW %>% group_by(Genotype,variable) %>%
    summarise(mean = mean(value),se = sd(value) / sqrt(n()))

ggplot(d, aes(x = variable, y = mean, fill = Genotype)) + 
  geom_bar(position = position_dodge(), stat = "identity", 
      colour="black", size=.3) +
  geom_errorbar(aes(ymin = mean - se, ymax = mean + se), 
      size=.3, width=.2, position=position_dodge(.9)) +
  xlab("Time") +
  ylab("Weight [g]") +
  scale_fill_hue(name = "Genotype", breaks = c("KO", "WT"), 
      labels = c("Knock-out", "Wild type")) +
  ggtitle("Effect of genotype on weight-gain") +
  scale_y_continuous(breaks = 0:20*4) +
  theme_bw()

Edit a specific Line of a Text File in C#

the easiest way is :

static void lineChanger(string newText, string fileName, int line_to_edit)
{
     string[] arrLine = File.ReadAllLines(fileName);
     arrLine[line_to_edit - 1] = newText;
     File.WriteAllLines(fileName, arrLine);
}

usage :

lineChanger("new content for this line" , "sample.text" , 34);

How can I change cols of textarea in twitter-bootstrap?

This works for me with twitter bootstrap 2 and simple_form 2.0.4
Result is a span6 text area in a span9 row

 <div class="row" >
   <div class="span9">
     <%= f.input :some_text, :input_html => {:rows => 5, :placeholder => "Enter some text.", :class => "span6"}%>
   </div>
 </div>

How to get the title of HTML page with JavaScript?

Can use getElementsByTagName

var x = document.getElementsByTagName("title")[0];

alert(x.innerHTML)

// or

alert(x.textContent)

// or

document.querySelector('title')

Edits as suggested by Paul

Get file version in PowerShell

Just another way to do it is to use the built-in file access technique:

(get-item .\filename.exe).VersionInfo | FL

You can also get any particular property off the VersionInfo, thus:

(get-item .\filename.exe).VersionInfo.FileVersion

This is quite close to the dir technique.

How to convert Windows end of line in Unix end of line (CR/LF to LF)

There should be a program called dos2unix that will fix line endings for you. If it's not already on your Linux box, it should be available via the package manager.

How to Remove Line Break in String

just this:

str = Replace(str, vbCrLf, "")

Windows Forms - Enter keypress activates submit button?

The Form has a KeyPreview property that you can use to intercept the keypress.

Using StringWriter for XML Serialization

One problem with StringWriter is that by default it doesn't let you set the encoding which it advertises - so you can end up with an XML document advertising its encoding as UTF-16, which means you need to encode it as UTF-16 if you write it to a file. I have a small class to help with that though:

public sealed class StringWriterWithEncoding : StringWriter
{
    public override Encoding Encoding { get; }

    public StringWriterWithEncoding (Encoding encoding)
    {
        Encoding = encoding;
    }    
}

Or if you only need UTF-8 (which is all I often need):

public sealed class Utf8StringWriter : StringWriter
{
    public override Encoding Encoding => Encoding.UTF8;
}

As for why you couldn't save your XML to the database - you'll have to give us more details about what happened when you tried, if you want us to be able to diagnose/fix it.

Passing just a type as a parameter in C#

Use generic types !

  class DataExtraction<T>
{
    DateRangeReport dateRange;
    List<Predicate> predicates;
    List<string> cids;

    public DataExtraction( DateRangeReport dateRange,
                           List<Predicate> predicates,
                           List<string> cids)            

    {
        this.dateRange = dateRange;
        this.predicates = predicates;
        this.cids = cids;
    }
}

And call it like this :

  DataExtraction<AdPerformanceRow> extractor = new DataExtraction<AdPerformanceRow>(dates, predicates , cids);

creating Hashmap from a JSON String

consider this json string

{
    "12": [
        {
            "dash_url": "http://mediaserver.superprofs.com:1935/vods3/_definst_/mp4:amazons3/superprofs-media/private/lectures/12/12_960x540_200k.mp4/manifest.mpd",
            "video_bitrate": "200k",
            "audio_bitrate": "32k",
            "video_width": 960,
            "video_height": 540,
            "file_size": 125465600
        },
        {
            "dash_url": "http://mediaserver.superprofs.com:1935/vods3/_definst_/mp4:amazons3/superprofs-media/private/lectures/12/12_960x540_80k.mp4/manifest.mpd",
            "video_bitrate": "80k",
            "audio_bitrate": "32k",
            "video_width": 960,
            "video_height": 540,
            "file_size": 50186240
        },
        {
            "dash_url": "http://mediaserver.superprofs.com:1935/vods3/_definst_/mp4:amazons3/superprofs-media/private/lectures/12/12_640x360_201k.mp4/manifest.mpd",
            "video_bitrate": "201k",
            "audio_bitrate": "32k",
            "video_width": 640,
            "video_height": 360,
            "file_size": 145934731
        },
        {
            "dash_url": "http://mediaserver.superprofs.com:1935/vods3/_definst_/mp4:amazons3/superprofs-media/private/lectures/12/12_640x360_199k.mp4/manifest.mpd",
            "video_bitrate": "199k",
            "audio_bitrate": "32k",
            "video_width": 640,
            "video_height": 360,
            "file_size": 145800030
        },
        {
            "dash_url": "http://mediaserver.superprofs.com:1935/vods3/_definst_/mp4:amazons3/superprofs-media/private/lectures/12/12_640x360_79k.mp4/manifest.mpd",
            "video_bitrate": "79k",
            "audio_bitrate": "32k",
            "video_width": 640,
            "video_height": 360,
            "file_size": 71709477
        }
    ],
    "13": [
        {
            "dash_url": "http://mediaserver.superprofs.com:1935/vods3/_definst_/mp4:amazons3/superprofs-media/private/lectures/13/13_960x540_200k.mp4/manifest.mpd",
            "video_bitrate": "200k",
            "audio_bitrate": "32k",
            "video_width": 960,
            "video_height": 540,
            "file_size": 172902400
        },
        {
            "dash_url": "http://mediaserver.superprofs.com:1935/vods3/_definst_/mp4:amazons3/superprofs-media/private/lectures/13/13_960x540_80k.mp4/manifest.mpd",
            "video_bitrate": "80k",
            "audio_bitrate": "32k",
            "video_width": 960,
            "video_height": 540,
            "file_size": 69160960
        },
        {
            "dash_url": "http://mediaserver.superprofs.com:1935/vods3/_definst_/mp4:amazons3/superprofs-media/private/lectures/13/13_640x360_201k.mp4/manifest.mpd",
            "video_bitrate": "201k",
            "audio_bitrate": "32k",
            "video_width": 640,
            "video_height": 360,
            "file_size": 199932081
        },
        {
            "dash_url": "http://mediaserver.superprofs.com:1935/vods3/_definst_/mp4:amazons3/superprofs-media/private/lectures/13/13_640x360_199k.mp4/manifest.mpd",
            "video_bitrate": "199k",
            "audio_bitrate": "32k",
            "video_width": 640,
            "video_height": 360,
            "file_size": 199630781
        },
        {
            "dash_url": "http://mediaserver.superprofs.com:1935/vods3/_definst_/mp4:amazons3/superprofs-media/private/lectures/13/13_640x360_79k.mp4/manifest.mpd",
            "video_bitrate": "79k",
            "audio_bitrate": "32k",
            "video_width": 640,
            "video_height": 360,
            "file_size": 98303415
        }
    ],
    "14": [
        {
            "dash_url": "http://mediaserver.superprofs.com:1935/vods3/_definst_/mp4:amazons3/superprofs-media/private/lectures/14/14_960x540_200k.mp4/manifest.mpd",
            "video_bitrate": "200k",
            "audio_bitrate": "32k",
            "video_width": 960,
            "video_height": 540,
            "file_size": 205747200
        },
        {
            "dash_url": "http://mediaserver.superprofs.com:1935/vods3/_definst_/mp4:amazons3/superprofs-media/private/lectures/14/14_960x540_80k.mp4/manifest.mpd",
            "video_bitrate": "80k",
            "audio_bitrate": "32k",
            "video_width": 960,
            "video_height": 540,
            "file_size": 82298880
        },
        {
            "dash_url": "http://mediaserver.superprofs.com:1935/vods3/_definst_/mp4:amazons3/superprofs-media/private/lectures/14/14_640x360_201k.mp4/manifest.mpd",
            "video_bitrate": "201k",
            "audio_bitrate": "32k",
            "video_width": 640,
            "video_height": 360,
            "file_size": 237769546
        },
        {
            "dash_url": "http://mediaserver.superprofs.com:1935/vods3/_definst_/mp4:amazons3/superprofs-media/private/lectures/14/14_640x360_199k.mp4/manifest.mpd",
            "video_bitrate": "199k",
            "audio_bitrate": "32k",
            "video_width": 640,
            "video_height": 360,
            "file_size": 237395552
        },
        {
            "dash_url": "http://mediaserver.superprofs.com:1935/vods3/_definst_/mp4:amazons3/superprofs-media/private/lectures/14/14_640x360_79k.mp4/manifest.mpd",
            "video_bitrate": "79k",
            "audio_bitrate": "32k",
            "video_width": 640,
            "video_height": 360,
            "file_size": 116885686
        }
    ],
    "15": [
        {
            "dash_url": "http://mediaserver.superprofs.com:1935/vods3/_definst_/mp4:amazons3/superprofs-media/private/lectures/15/15_960x540_200k.mp4/manifest.mpd",
            "video_bitrate": "200k",
            "audio_bitrate": "32k",
            "video_width": 960,
            "video_height": 540,
            "file_size": 176128000
        },
        {
            "dash_url": "http://mediaserver.superprofs.com:1935/vods3/_definst_/mp4:amazons3/superprofs-media/private/lectures/15/15_960x540_80k.mp4/manifest.mpd",
            "video_bitrate": "80k",
            "audio_bitrate": "32k",
            "video_width": 960,
            "video_height": 540,
            "file_size": 70451200
        },
        {
            "dash_url": "http://mediaserver.superprofs.com:1935/vods3/_definst_/mp4:amazons3/superprofs-media/private/lectures/15/15_640x360_201k.mp4/manifest.mpd",
            "video_bitrate": "201k",
            "audio_bitrate": "32k",
            "video_width": 640,
            "video_height": 360,
            "file_size": 204263286
        },
        {
            "dash_url": "http://mediaserver.superprofs.com:1935/vods3/_definst_/mp4:amazons3/superprofs-media/private/lectures/15/15_640x360_199k.mp4/manifest.mpd",
            "video_bitrate": "199k",
            "audio_bitrate": "32k",
            "video_width": 640,
            "video_height": 360,
            "file_size": 204144447
        },
        {
            "dash_url": "http://mediaserver.superprofs.com:1935/vods3/_definst_/mp4:amazons3/superprofs-media/private/lectures/15/15_640x360_79k.mp4/manifest.mpd",
            "video_bitrate": "79k",
            "audio_bitrate": "32k",
            "video_width": 640,
            "video_height": 360,
            "file_size": 100454382
        }
    ]
}

using jackson parser

 private static ObjectMapper underScoreToCamelCaseMapper;

    static {
        final DateFormat df = new SimpleDateFormat("yyyy-MM-dd HH:mm:ss");

        underScoreToCamelCaseMapper = new ObjectMapper();
        underScoreToCamelCaseMapper.setDateFormat(df);
        underScoreToCamelCaseMapper.configure(DeserializationFeature.FAIL_ON_UNKNOWN_PROPERTIES, false);
        underScoreToCamelCaseMapper.setVisibility(PropertyAccessor.FIELD, JsonAutoDetect.Visibility.ANY);
        underScoreToCamelCaseMapper.setPropertyNamingStrategy(
                PropertyNamingStrategy.CAMEL_CASE_TO_LOWER_CASE_WITH_UNDERSCORES);
}
    public static <T> T parseUnderScoredResponse(String json, Class<T> classOfT) {
        try {
            if (json == null) {
                return null;
            }
            return underScoreToCamelCaseMapper.readValue(json, classOfT);

        } catch (JsonParseException e) {
        } catch (JsonMappingException e) {
        } catch (IOException e) {
        }
        return null;
    }

use following code to parse

 HashMap<String, ArrayList<Video>> integerArrayListHashMap =
                JsonHandler.parseUnderScoredResponse(test, MyHashMap.class);

where MyHashMap is

 private static class MyHashMap extends HashMap<String,ArrayList<Video>>{
    }

Single selection in RecyclerView

Please try this... This works for me..

In adapter,take a sparse boolean array.

SparseBooleanArray sparseBooleanArray;

In constructor initialise this,

   sparseBooleanArray=new SparseBooleanArray();

In bind holder add,

@Override
public void onBindViewHolder(DispositionViewHolder holder, final int position) {
   holder.tv_disposition.setText(dispList.get(position).getName());
    if(sparseBooleanArray.get(position,false))
    {
        holder.rd_disp.setChecked(true);
    }
    else
    {
        holder.rd_disp.setChecked(false);


    }
    setClickListner(holder,position);
}

private void setClickListner(final DispositionViewHolder holder, final int position) {
    holder.rd_disp.setOnClickListener(new View.OnClickListener() {
        @Override
        public void onClick(View v) {
            sparseBooleanArray.clear();
            sparseBooleanArray.put(position, true);
            notifyDataSetChanged();


        }
    });
}

rd_disp is radio button in xml file.

So when the recycler view load the items,in bindView Holder it check whether the sparseBooleanArray contain the value "true" correspnding to its position.

If the value returned is true then we set the radio button selection true.Else we set the selection false. In onclickHolder I have cleared the sparseArray and set the value true corresponding to that position. When I call notify datasetChange it again call the onBindViewHolder and the condition are checked again. This makes our selection to only select particular radio.

Convert List(of object) to List(of string)

If you want more control over how the conversion takes place, you can use ConvertAll:

var stringList = myList.ConvertAll(obj => obj.SomeToStringMethod());

How to make String.Contains case insensitive?

You can use:

if (myString1.IndexOf("AbC", StringComparison.OrdinalIgnoreCase) >=0) {
    //...
}

This works with any .NET version.

Python loop for inside lambda

To add on to chepner's answer for Python 3.0 you can alternatively do:

x = lambda x: list(map(print, x))

Of course this is only if you have the means of using Python > 3 in the future... Looks a bit cleaner in my opinion, but it also has a weird return value, but you're probably discarding it anyway.

I'll just leave this here for reference.

Iterating through array - java

If you are using an array (and purely an array), the lookup of "contains" is O(N), because worst case, you must iterate the entire array. Now if the array is sorted you can use a binary search, which reduces the search time to log(N) with the overhead of the sort.

If this is something that is invoked repeatedly, place it in a function:

private boolean inArray(int[] array, int value)
{  
     for (int i = 0; i < array.length; i++)
     {
        if (array[i] == value) 
        {
            return true;
        }
     }
    return false;  
}  

Using SSIS BIDS with Visual Studio 2012 / 2013

Today March 6, 2013, Microsoft released SQL Server Data Tools – Business Intelligence for Visual Studio 2012 (SSDT BI) templates. With SSDT BI for Visual Studio 2012 you can develop and deploy SQL Server Business intelligence projects. Projects created in Visual Studio 2010 can be opened in Visual Studio 2012 and the other way around without upgrading or downgrading – it just works.

The download/install is named to ensure you get the SSDT templates that contain the Business Intelligence projects. The setup for these tools is now available from the web and can be downloaded in multiple languages right here: http://www.microsoft.com/download/details.aspx?id=36843

What is a stack pointer used for in microprocessors?

Should you ever crave deeper understanding, I heartily recommend Patterson and Hennessy as an intro and Hennessy and Patterson as an intermediate to advanced text. They're pricey, but truly non-pareil; I just wish either or both were available when I got my Masters' degree and entered the workforce designing chips, systems, and parts of system software for them (but, alas!, that was WAY too long ago;-). Stack pointers are so crucial (and the distinction between a microprocessor and any other kind of CPU so utterly meaningful in this context... or, for that matter, in ANY other context, in the last few decades...!-) that I doubt anything but a couple of thorough from-the-ground-up refreshers can help!-)

Angular2 - Radio Button Binding

use [value]="1" instead of value="1"

<input name="options" ng-control="options" type="radio" [value]="1"  [(ngModel)]="model.options" ><br/>

<input name="options" ng-control="options" type="radio" [value]="2" [(ngModel)]="model.options" ><br/>

Edit:

As suggested by thllbrg "For angular 2.1+ use [(ngModel)] instead of [(ng-model)] "

How to define constants in ReactJS

Warning: this is an experimental feature that could dramatically change or even cease to exist in future releases

You can use ES7 statics:

npm install babel-preset-stage-0

And then add "stage-0" to .babelrc presets:

{
    "presets": ["es2015", "react", "stage-0"]
}

Afterwards, you go

class Component extends React.Component {
    static foo = 'bar';
    static baz = {a: 1, b: 2}
}

And then you use them like this:

Component.foo

How to print / echo environment variables?

This works too, with the semi-colon.

NAME=sam; echo $NAME

How do implement a breadth first traversal?

For implementing the breadth first search, you should use a queue. You should push the children of a node to the queue (left then right) and then visit the node (print data). Then, yo should remove the node from the queue. You should continue this process till the queue becomes empty. You can see my implementation of the BFS here: https://github.com/m-vahidalizadeh/foundations/blob/master/src/algorithms/TreeTraverse.java

How do I make the text box bigger in HTML/CSS?

Try this:

#signin input {
    background-color:#FFF;
    height: 1.5em;
    /* or */
    line-height: 1.5em;
}

swift 3.0 Data to String?

for swift 5

let testString = "This is a test string"
let somedata = testString.data(using: String.Encoding.utf8)
let backToString = String(data: somedata!, encoding: String.Encoding.utf8) as String?
print("testString > \(testString)")
//testString > This is a test string
print("somedata > \(String(describing: somedata))")
//somedata > Optional(21 bytes)
print("backToString > \(String(describing: backToString))")
//backToString > Optional("This is a test string")

Javascript change color of text and background to input value

Depending on which event you actually want to use (textbox change, or button click), you can try this:

HTML:

<input id="color" type="text" onchange="changeBackground(this);" />
<br />
<span id="coltext">This text should have the same color as you put in the text box</span>

JS:

function changeBackground(obj) {
    document.getElementById("coltext").style.color = obj.value;
}

DEMO: http://jsfiddle.net/6pLUh/

One minor problem with the button was that it was a submit button, in a form. When clicked, that submits the form (which ends up just reloading the page) and any changes from JavaScript are reset. Just using the onchange allows you to change the color based on the input.

How to write a switch statement in Ruby

In Ruby 2.0, you can also use lambdas in case statements, as follows:

is_even = ->(x) { x % 2 == 0 }

case number
when 0 then puts 'zero'
when is_even then puts 'even'
else puts 'odd'
end

You can also create your own comparators easily using a Struct with a custom ===

Moddable = Struct.new(:n) do
  def ===(numeric)
    numeric % n == 0
  end
end

mod4 = Moddable.new(4)
mod3 = Moddable.new(3)

case number
when mod4 then puts 'multiple of 4'
when mod3 then puts 'multiple of 3'
end

(Example taken from "Can procs be used with case statements in Ruby 2.0?".)

Or, with a complete class:

class Vehicle
  def ===(another_vehicle)
    self.number_of_wheels == another_vehicle.number_of_wheels
  end
end

four_wheeler = Vehicle.new 4
two_wheeler = Vehicle.new 2

case vehicle
when two_wheeler
  puts 'two wheeler'
when four_wheeler
  puts 'four wheeler'
end

(Example taken from "How A Ruby Case Statement Works And What You Can Do With It".)

Error Code: 1290. The MySQL server is running with the --secure-file-priv option so it cannot execute this statement

I had to set

C:\ProgramData\MySQL\MySQL Server 8.0/my.ini  secure-file-priv=""

When I commented line with secure-file-priv, secure-file-priv was null and I couldn't download data.

Bootstrap onClick button event

There is no show event in js - you need to bind your button either to the click event:

$('#id').on('click', function (e) {

     //your awesome code here

})

Mind that if your button is inside a form, you may prefer to bind the whole form to the submit event.

Get RETURN value from stored procedure in SQL

The accepted answer is invalid with the double EXEC (only need the first EXEC):

DECLARE @returnvalue int;
EXEC @returnvalue = SP_SomeProc
PRINT @returnvalue

And you still need to call PRINT (at least in Visual Studio).

How to auto resize and adjust Form controls with change in resolution

Use Dock and Anchor properties. Here is a good article. Note that these will handle changes when maximizing/minimizing. That is a little different that if the screen resolution changes, but it will be along the same idea.

Use CSS to remove the space between images

Make them display: block in your CSS.

How to truncate the time on a DateTime object in Python?

There is a module datetime_truncate which handlers this for you. It just calls datetime.replace.

How to make GREP select only numeric values?

No need to used grep here, Try this:

df . -B MB | tail -1 | awk {'print substr($5, 1, length($5)-1)'}

I want to remove double quotes from a String

If you only want to remove quotes from the beginning or the end, use the following regular expression:

'"Hello"'.replace(/(^"|"$)/g, '');

Difference between h:button and h:commandButton

h:button - clicking on a h:button issues a bookmarkable GET request.

h:commandbutton - Instead of a get request, h:commandbutton issues a POST request which sends the form data back to the server.

Rails has_many with alias name

To complete @SamSaffron's answer :

You can use class_name with either foreign_key or inverse_of. I personally prefer the more abstract declarative, but it's really just a matter of taste :

class BlogPost
  has_many :images, class_name: "BlogPostImage", inverse_of: :blog_post  
end

and you need to make sure you have the belongs_to attribute on the child model:

class BlogPostImage
  belongs_to :blog_post
end

Convert boolean to int in Java

int val = b? 1 : 0;

How to debug ORA-01775: looping chain of synonyms?

Try this select to find the problematic synonyms, it lists all synonyms that are pointing to an object that does not exist (tables,views,sequences,packages, procedures, functions)

SELECT *
FROM dba_synonyms
WHERE table_owner = 'USER'
    AND (
        NOT EXISTS (
            SELECT *
            FROM dba_tables
            WHERE dba_synonyms.table_name = dba_tables.TABLE_NAME
            )
        AND NOT EXISTS (
            SELECT *
            FROM dba_views
            WHERE dba_synonyms.table_name = dba_views.VIEW_NAME
            )
        AND NOT EXISTS (
            SELECT *
            FROM dba_sequences
            WHERE dba_synonyms.table_name = dba_sequences.sequence_NAME
            )
        AND NOT EXISTS (
            SELECT *
            FROM dba_dependencies
            WHERE type IN (
                    'PACKAGE'
                    ,'PROCEDURE'
                    ,'FUNCTION'
                    )
                AND dba_synonyms.table_name = dba_dependencies.NAME
            )
        )

Login to website, via C#

You can simplify things quite a bit by creating a class that derives from WebClient, overriding its GetWebRequest method and setting a CookieContainer object on it. If you always set the same CookieContainer instance, then cookie management will be handled automatically for you.

But the only way to get at the HttpWebRequest before it is sent is to inherit from WebClient and override that method.

public class CookieAwareWebClient : WebClient
{
    private CookieContainer cookie = new CookieContainer();

    protected override WebRequest GetWebRequest(Uri address)
    {
        WebRequest request = base.GetWebRequest(address);
        if (request is HttpWebRequest)
        {
            (request as HttpWebRequest).CookieContainer = cookie;
        }
        return request;
    }
}

var client = new CookieAwareWebClient();
client.BaseAddress = @"https://www.site.com/any/base/url/";
var loginData = new NameValueCollection();
loginData.Add("login", "YourLogin");
loginData.Add("password", "YourPassword");
client.UploadValues("login.php", "POST", loginData);

//Now you are logged in and can request pages    
string htmlSource = client.DownloadString("index.php");

Why does "pip install" inside Python raise a SyntaxError?

Programmatically, the following currently works. I see all the answers post 10.0 and all, but none of them are the correct path for me. Within Kaggle for sure, this apporach works

from pip._internal import main as _main

package_names=['pandas'] #packages to install
_main(['install'] + package_names + ['--upgrade']) 

Apache SSL Configuration Error (SSL Connection Error)

Similar to other answers, this error can be experienced when there are no sites configured to use SSL.

I had the error when I upgraded from Debian Wheezy to Debian Jessie. The new version of Apache requires a site configuration file ending in .conf. Because my configuration file didn't, it was being ignored, and there were no others configured to serve SSL connections.

Where to find 64 bit version of chromedriver.exe for Selenium WebDriver?

You will find newest version of the chromedriver here: http://chromedriver.storage.googleapis.com/index.html - there is a 64bit version for linux.

How can I check what version/edition of Visual Studio is installed programmatically?

An updated answer to this question would be the following :

"C:\Program Files (x86)\Microsoft Visual Studio\Installer\vswhere.exe" -latest -property productId

Resolves to 2019

"C:\Program Files (x86)\Microsoft Visual Studio\Installer\vswhere.exe" -latest -property catalog_productLineVersion

Resolves to Microsoft.VisualStudio.Product.Professional

ERROR: Sonar server 'http://localhost:9000' can not be reached

In the config file there is a colon instead of an equal sign after the sonar.web.host.

Is:

sonar.web.host:sonarqube

Should be

sonar.web.host=sonarqube

Android getText from EditText field

Try out this will solve ur problem ....

EditText etxt = (EditText)findviewbyid(R.id.etxt);
String str_value = etxt.getText().toString();

Difference between RegisterStartupScript and RegisterClientScriptBlock?

Here's a simplest example from ASP.NET Community, this gave me a clear understanding on the concept....

what difference does this make?

For an example of this, here is a way to put focus on a text box on a page when the page is loaded into the browser—with Visual Basic using the RegisterStartupScript method:

Page.ClientScript.RegisterStartupScript(Me.GetType(), "Testing", _ 
"document.forms[0]['TextBox1'].focus();", True)

This works well because the textbox on the page is generated and placed on the page by the time the browser gets down to the bottom of the page and gets to this little bit of JavaScript.

But, if instead it was written like this (using the RegisterClientScriptBlock method):

Page.ClientScript.RegisterClientScriptBlock(Me.GetType(), "Testing", _
"document.forms[0]['TextBox1'].focus();", True)

Focus will not get to the textbox control and a JavaScript error will be generated on the page

The reason for this is that the browser will encounter the JavaScript before the text box is on the page. Therefore, the JavaScript will not be able to find a TextBox1.

How do you change text to bold in Android?

You can use this for font

create a Class Name TypefaceTextView and extend the TextView

private static Map mTypefaces;

public TypefaceTextView(final Context context) {
    this(context, null);
}

public TypefaceTextView(final Context context, final AttributeSet attrs) {
    this(context, attrs, 0);
}

public TypefaceTextView(final Context context, final AttributeSet attrs, final int defStyle) {
    super(context, attrs, defStyle);
    if (mTypefaces == null) {
        mTypefaces = new HashMap<String, Typeface>();
    }

    if (this.isInEditMode()) {
        return;
    }

    final TypedArray array = context.obtainStyledAttributes(attrs, styleable.TypefaceTextView);
    if (array != null) {
        final String typefaceAssetPath = array.getString(
                R.styleable.TypefaceTextView_customTypeface);

        if (typefaceAssetPath != null) {
            Typeface typeface = null;

            if (mTypefaces.containsKey(typefaceAssetPath)) {
                typeface = mTypefaces.get(typefaceAssetPath);
            } else {
                AssetManager assets = context.getAssets();
                typeface = Typeface.createFromAsset(assets, typefaceAssetPath);
                mTypefaces.put(typefaceAssetPath, typeface);
            }

            setTypeface(typeface);
        }
        array.recycle();
    }
}

paste the font in the fonts folder created in the asset folder

<packagename.TypefaceTextView
        android:layout_width="0dp"
        android:layout_height="match_parent"
        android:layout_weight="1.5"
        android:gravity="center"
        android:text="TRENDING TURFS"
        android:textColor="#000"
        android:textSize="20sp"
        app:customTypeface="fonts/pompiere.ttf" />**here pompiere.ttf is the font name**

Place the lines in the parent layout in the xml

 xmlns:app="http://schemas.android.com/apk/res/com.mediasters.wheresmyturf"
xmlns:custom="http://schemas.android.com/apk/res-auto"

Adding an img element to a div with javascript

It should be:

document.getElementById("placehere").appendChild(elem);

And place your div before your javascript, because if you don't, the javascript executes before the div exists. Or wait for it to load. So your code looks like this:

<html>

<body>
<script type="text/javascript">
window.onload=function(){
var elem = document.createElement("img");
elem.setAttribute("src", "http://img.zohostatic.com/discussions/v1/images/defaultPhoto.png");
elem.setAttribute("height", "768");
elem.setAttribute("width", "1024");
elem.setAttribute("alt", "Flower");
document.getElementById("placehere").appendChild(elem);
}
</script>
<div id="placehere">

</div>

</body>
</html>

To prove my point, see this with the onload and this without the onload. Fire up the console and you'll find an error stating that the div doesn't exist or cannot find appendChild method of null.

Is Spring annotation @Controller same as @Service?

If you look at the definitions of @Controller, @Service annotations, then you'll find that these are special type of @Component annotation.

@Component
public @interface Service {
    ….
}

 

@Component
public @interface Controller {
    …
}

So what's the difference?

@Controller

The @Controller annotation indicates that a particular class serves the role of a controller. The @Controller annotation acts as a stereotype for the annotated class, indicating its role.

What’s special about @Controller?

You cannot switch this annotation with any other like @Service or @Repository, even though they look same. The dispatcher scans the classes annotated with @Controller and detects @RequestMapping annotations within them. You can only use @RequestMapping on @Controller annotated classes.


@Service

@Services hold business logic and call method in repository layer.

What’s special about @Service?

Apart from the fact that it is used to indicate that it's holding the business logic, there’s no noticeable specialty that this annotation provides, but who knows, spring may add some additional exceptional in future.

Linked answer: What's the difference between @Component, @Repository & @Service annotations in Spring?

How to get the current time in milliseconds in C Programming

If you're on a Unix-like system, use gettimeofday and convert the result from microseconds to milliseconds.

Call a global variable inside module

Download the bootbox typings

Then add a reference to it inside your .ts file.

Adding a dictionary to another

Create an Extension Method most likely you will want to use this more than once and this prevents duplicate code.

Implementation:

 public static void AddRange<T, S>(this Dictionary<T, S> source, Dictionary<T, S> collection)
 {
        if (collection == null)
        {
            throw new ArgumentNullException("Collection is null");
        }

        foreach (var item in collection)
        {
            if(!source.ContainsKey(item.Key)){ 
               source.Add(item.Key, item.Value);
            }
            else
            {
               // handle duplicate key issue here
            }  
        } 
 }

Usage:

Dictionary<string,string> animals = new Dictionary<string,string>();
Dictionary<string,string> newanimals = new Dictionary<string,string>();

animals.AddRange(newanimals);

How to rename HTML "browse" button of an input type=file?

You can also use Uploadify, which is a great jQuery upload plugin, it let's you upload multiple files, and also style the file fields easily. http://www.uploadify.com

How to check whether particular port is open or closed on UNIX?

Try (maybe as root)

lsof -i -P

and grep the output for the port you are looking for.

For example to check for port 80 do

lsof -i -P | grep :80

How to set Java classpath in Linux?

You have to use ':' colon instead of ';' semicolon.

As it stands now you try to execute the jar file which has not the execute bit set, hence the Permission denied.

And the variable must be CLASSPATH not classpath.

How to run vi on docker container?

Your container probably haven't installed it out of the box.

Run apt-get install vim in the terminal and you should be ready to go.

How do you show animated GIFs on a Windows Form (c#)

I had the same problem. Whole form (including gif) stopping to redraw itself because of long operation working in the background. Here is how i solved this.

  private void MyThreadRoutine()
  {
   this.Invoke(this.ShowProgressGifDelegate);
   //your long running process
   System.Threading.Thread.Sleep(5000);
   this.Invoke(this.HideProgressGifDelegate);
  }

  private void button1_Click(object sender, EventArgs e)
  {
   ThreadStart myThreadStart = new ThreadStart(MyThreadRoutine);
   Thread myThread = new Thread(myThreadStart);
   myThread.Start(); 
  }

I simply created another thread to be responsible for this operation. Thanks to this initial form continues redrawing without problems (including my gif working). ShowProgressGifDelegate and HideProgressGifDelegate are delegates in form that set visible property of pictureBox with gif to true/false.

Best way to create a temp table with same columns and type as a permanent table

Clone Temporary Table Structure to New Physical Table in SQL Server

enter image description here

we will see how to Clone Temporary Table Structure to New Physical Table in SQL Server.This is applicable for both Azure SQL db and on-premises.

Demo SQL Script

IF OBJECT_ID('TempDB..#TempTable') IS NOT NULL
    DROP TABLE #TempTable;

SELECT 1 AS ID,'Arul' AS Names
INTO
#TempTable;

SELECT * FROM #TempTable;

METHOD 1

SELECT * INTO TempTable1 FROM #TempTable WHERE 1=0;

EXEC SP_HELP TempTable1;

enter image description here

METHOD 2

SELECT TOP 0 * INTO TempTable1 FROM #TempTable;

EXEC SP_HELP TempTable1;

enter image description here

Laravel check if collection is empty

I prefer

(!$mentor)

Is more effective and accurate

Calendar.getInstance(TimeZone.getTimeZone("UTC")) is not returning UTC time

You are definitely missing a small thing and that is you are not setting a default value:

TimeZone.setDefault(TimeZone.getTimeZone("UTC"));

So the code would look like:

TimeZone.setDefault(TimeZone.getTimeZone("UTC"));
Calendar cal_Two = Calendar.getInstance(TimeZone.getTimeZone("UTC"));
System.out.println(cal_Two.getTime());

Explanation: If you want to change the time zone, set the default time zone using TimeZone.setDefault()

Making the Android emulator run faster

I hope this will help you.

Goto to your BIOS settings. Enable your Virtualization technology in your settings..

It solved my problem...

PHP Fatal error: Cannot access empty property

I realise this answer is not a direct response to the problem described by the OP, but I found this question as a result of searching for the same error message. I thought it worth posting my experience here just in case anybody is muddling over the same thing...

You can encounter the error in question as a result of a poorly formatted for loop over an associative array. In a fit of bone-headedness, I was using -> instead of => in my for statement:

        foreach ($object->someArray as $key->$val) {
            // do something
        }

Of course, I should have had:

        foreach ($object->someArray as $key=>$val) {
            // do something
        }

I confused myself at first, thinking the reported error was referring to the someArray property!

Google maps API V3 method fitBounds()

I have the same problem that you describe although I'm building up my LatLngBounds as proposed by above. The problem is that things are async and calling map.fitBounds() at the wrong time may leave you with a result like in the Q. The best way I found is to place the call in an idle handler like this:

google.maps.event.addListenerOnce(map, 'idle', function() {
    map.fitBounds(markerBounds);
});

Rotation of 3D vector?

I needed to rotate a 3D model around one of the three axes {x, y, z} in which that model was embedded and this was the top result for a search of how to do this in numpy. I used the following simple function:

def rotate(X, theta, axis='x'):
  '''Rotate multidimensional array `X` `theta` degrees around axis `axis`'''
  c, s = np.cos(theta), np.sin(theta)
  if axis == 'x': return np.dot(X, np.array([
    [1.,  0,  0],
    [0 ,  c, -s],
    [0 ,  s,  c]
  ]))
  elif axis == 'y': return np.dot(X, np.array([
    [c,  0,  -s],
    [0,  1,   0],
    [s,  0,   c]
  ]))
  elif axis == 'z': return np.dot(X, np.array([
    [c, -s,  0 ],
    [s,  c,  0 ],
    [0,  0,  1.],
  ]))

How can I get System variable value in Java?

As mentioned by sombody above, restarting eclipse worked for me for the user defined environment variable. After I restart eclipse IDE, System.getenv() is picking up my environment variable.

SQL providerName in web.config

 WebConfigurationManager.ConnectionStrings["YourConnectionString"].ProviderName;

HTML5 iFrame Seamless Attribute

iO8 has removed support for the iframe seamless attribute.

  • Tested in Safari, HomeScreen, new WKWebView and UIWebView.

Full Details and performance review of other iOS 8 changes:

Inserting a Python datetime.datetime object into MySQL

when iserting into t-sql

this fails:

select CONVERT(datetime,'2019-09-13 09:04:35.823312',21)

this works:

select CONVERT(datetime,'2019-09-13 09:04:35.823',21)

easy way:

regexp = re.compile(r'\.(\d{6})')
def to_splunk_iso(dt):
    """Converts the datetime object to Splunk isoformat string."""
    # 6-digits string.
    microseconds = regexp.search(dt).group(1)
    return regexp.sub('.%d' % round(float(microseconds) / 1000), dt)

How can I create a marquee effect?

With a small change of the markup, here's my approach (I've just inserted a span inside the paragraph):

_x000D_
_x000D_
.marquee {
  width: 450px;
  margin: 0 auto;
  overflow: hidden;
  box-sizing: border-box;
}

.marquee span {
  display: inline-block;
  width: max-content;

  padding-left: 100%;
  /* show the marquee just outside the paragraph */
  will-change: transform;
  animation: marquee 15s linear infinite;
}

.marquee span:hover {
  animation-play-state: paused
}


@keyframes marquee {
  0% { transform: translate(0, 0); }
  100% { transform: translate(-100%, 0); }
}


/* Respect user preferences about animations */

@media (prefers-reduced-motion: reduce) {
  .marquee span {
    animation-iteration-count: 1;
    animation-duration: 0.01; 
    /* instead of animation: none, so an animationend event is 
     * still available, if previously attached.
     */
    width: auto;
    padding-left: 0;
  }
}
_x000D_
<p class="marquee">
   <span>
       When I had journeyed half of our life's way, I found myself
       within a shadowed forest, for I had lost the path that 
       does not stray. – (Dante Alighieri, <i>Divine Comedy</i>. 
       1265-1321)
   </span>
   </p>
_x000D_
_x000D_
_x000D_


No hardcoded values — dependent on paragraph width — have been inserted.

The animation applies the CSS3 transform property (use prefixes where needed) so it performs well.

If you need to insert a delay just once at the beginning then also set an animation-delay. If you need instead to insert a small delay at every loop then try to play with an higher padding-left (e.g. 150%)

How to change the link color in a specific class for a div CSS

smaller-size version:

#register a:link {color: #fff}

is there any alternative for ng-disabled in angular2?

Yes You can either set [disabled]= "true" or if it is an radio button or checkbox then you can simply use disable

For radio button:

<md-radio-button disabled>Select color</md-radio-button>

For dropdown:

<ng-select (selected)="someFunction($event)" [disabled]="true"></ng-select>

jquery ui Dialog: cannot call methods on dialog prior to initialization

Try this instead

$(document).ready(function() {
  $("#divDialog").dialog(opt).dialog("open");
});

You can also do:

var theDialog = $("#divDialog").dialog(opt);
theDialog.dialog("open");

That's because the dialog is not stored in $('#divDialog'), but on a new div that is created on the fly and returned by the .dialog(opt) function.

How to make the python interpreter correctly handle non-ASCII characters in string operations?

For what it was worth, my character set was utf-8 and I had included the classic "# -*- coding: utf-8 -*-" line.

However, I discovered that I didn't have Universal Newlines when reading this data from a webpage.

My text had two words, separated by "\r\n". I was only splitting on the \n and replacing the "\n".

Once I looped through and saw the character set in question, I realized the mistake.

So, it could also be within the ASCII character set, but a character that you didn't expect.

jquery how to use multiple ajax calls one after the end of the other

This is the most elegant solution I've been using for a while. It doesn't require external counter variable and it provides nice degree of encapsulation.

var urls = ['http://..', 'http://..', ..];

function ajaxRequest (urls) {
    if (urls.length > 0) {
        $.ajax({
            method: 'GET',
            url: urls.pop()
        })
        .done(function (result)) {
            ajaxRequest(urls);
        });
    }
}

ajaxRequest(urls); 

Reverse a string without using reversed() or [::-1]?

def reverse(s):
    return "".join(s[i] for i in range(len(s)-1, -1, -1))

What does %s and %d mean in printf in the C language?

"%s%d%s%d\n" is the format string; it tells the printf function how to format and display the output. Anything in the format string that doesn't have a % immediately in front of it is displayed as is.

%s and %d are conversion specifiers; they tell printf how to interpret the remaining arguments. %s tells printf that the corresponding argument is to be treated as a string (in C terms, a 0-terminated sequence of char); the type of the corresponding argument must be char *. %d tells printf that the corresponding argument is to be treated as an integer value; the type of the corresponding argument must be int. Since you're coming from a Java background, it's important to note that printf (like other variadic functions) is relying on you to tell it what the types of the remaining arguments are. If the format string were "%d%s%d%s\n", printf would attempt to treat "Length of string" as an integer value and i as a string, with tragic results.

Android appcompat v7:23

Original answer:

I too tried to change the support library to "23". When I changed the targetSdkVersion to 23, Android Studio reported the following error:

This support library should not use a lower version (22) than the targetSdkVersion (23)

I simply changed:

compile 'com.android.support:appcompat-v7:23.0.0'

to

compile 'com.android.support:appcompat-v7:+'

Although this fixed my issue, you should not use dynamic versions. After a few hours the new support repository was available and it is currently 23.0.1.


Pro tip:

You can use double quotes and create a ${supportLibVersion} variable for simplicity. Example:

ext {
    supportLibVersion = '23.1.1'
}

compile "com.android.support:appcompat-v7:${supportLibVersion}"
compile "com.android.support:design:${supportLibVersion}"
compile "com.android.support:palette-v7:${supportLibVersion}"
compile "com.android.support:customtabs:${supportLibVersion}"
compile "com.android.support:gridlayout-v7:${supportLibVersion}"

source: https://twitter.com/manidesto/status/669195097947377664

How do I get the name of the active user via the command line in OS X?

getting username in MAC terminal is easy...

I generally use whoami in terminal...

For example, in this case, I needed that to install Tomcat Server...

enter image description here

SQL Server Jobs with SSIS packages - Failed to decrypt protected XML node "DTS:Password" with error 0x8009000B

Little late to the game but i found a way to fix this for me that i had not seen anywhere else. Select your connection from Connection Managers. On the right you should see properties. Check to see if there are any expressions there if not add one. In your package explorer add a variable called connection to sql or whatever. Set the variable as a string and set the value as your connection string and include the User Id and password. Back to the connection manager properties and expression. From the drop down select ConnectionString and set the second box as the name of your variable. It should look like this

enter image description here

I could not for the life of me find another solution but this worked!

Unable to cast object of type 'System.DBNull' to type 'System.String`

A shorter form can be used:

return (accountNumber == DBNull.Value) ? string.Empty : accountNumber.ToString()

EDIT: Haven't paid attention to ExecuteScalar. It does really return null if the field is absent in the return result. So use instead:

return (accountNumber == null) ? string.Empty : accountNumber.ToString() 

Create 3D array using Python

There are many ways to address your problem.

  1. First one as accepted answer by @robert. Here is the generalised solution for it:
def multi_dimensional_list(value, *args):
  #args dimensions as many you like. EG: [*args = 4,3,2 => x=4, y=3, z=2]
  #value can only be of immutable type. So, don't pass a list here. Acceptable value = 0, -1, 'X', etc.
  if len(args) > 1:
    return [ multi_dimensional_list(value, *args[1:]) for col in range(args[0])]
  elif len(args) == 1: #base case of recursion
    return [ value for col in range(args[0])]
  else: #edge case when no values of dimensions is specified.
    return None

Eg:

>>> multi_dimensional_list(-1, 3, 4)  #2D list
[[-1, -1, -1, -1], [-1, -1, -1, -1], [-1, -1, -1, -1]]
>>> multi_dimensional_list(-1, 4, 3, 2)  #3D list
[[[-1, -1], [-1, -1], [-1, -1]], [[-1, -1], [-1, -1], [-1, -1]], [[-1, -1], [-1, -1], [-1, -1]], [[-1, -1], [-1, -1], [-1, -1]]]
>>> multi_dimensional_list(-1, 2, 3, 2, 2 )  #4D list
[[[[-1, -1], [-1, -1]], [[-1, -1], [-1, -1]], [[-1, -1], [-1, -1]]], [[[-1, -1], [-1, -1]], [[-1, -1], [-1, -1]], [[-1, -1], [-1, -1]]]]

P.S If you are keen to do validation for correct values for args i.e. only natural numbers, then you can write a wrapper function before calling this function.

  1. Secondly, any multidimensional dimensional array can be written as single dimension array. This means you don't need a multidimensional array. Here are the function for indexes conversion:
def convert_single_to_multi(value, max_dim):
  dim_count = len(max_dim)
  values = [0]*dim_count
  for i in range(dim_count-1, -1, -1): #reverse iteration
    values[i] = value%max_dim[i]
    value /= max_dim[i]
  return values


def convert_multi_to_single(values, max_dim):
  dim_count = len(max_dim)
  value = 0
  length_of_dimension = 1
  for i in range(dim_count-1, -1, -1): #reverse iteration
    value += values[i]*length_of_dimension
    length_of_dimension *= max_dim[i]
  return value

Since, these functions are inverse of each other, here is the output:

>>> convert_single_to_multi(convert_multi_to_single([1,4,6,7],[23,45,32,14]),[23,45,32,14])
[1, 4, 6, 7]
>>> convert_multi_to_single(convert_single_to_multi(21343,[23,45,32,14]),[23,45,32,14])
21343
  1. If you are concerned about performance issues then you can use some libraries like pandas, numpy, etc.

Tomcat Server Error - Port 8080 already in use

If you want to regain the 8080 port number you do so by opening the task manager and then process tab, right click java.exe process and click on end process as shown in image attached.

screen shot

Disable HTTP OPTIONS, TRACE, HEAD, COPY and UNLOCK methods in IIS

This one disables all bogus verbs and only allows GET and POST

<system.webServer>
  <security>
    <requestFiltering>
      <verbs allowUnlisted="false">
    <clear/>
    <add verb="GET" allowed="true"/>
    <add verb="POST" allowed="true"/>
      </verbs>
    </requestFiltering>
  </security>
</system.webServer>

The most efficient way to remove first N elements in a list?

l = [1, 2, 3, 4, 5]
del l[0:3] # Here 3 specifies the number of items to be deleted.

This is the code if you want to delete a number of items from the list. You might as well skip the zero before the colon. It does not have that importance. This might do as well.

l = [1, 2, 3, 4, 5]
del l[:3] # Here 3 specifies the number of items to be deleted.

How to enable local network users to access my WAMP sites?

if you use Windows and if you do all comments in above ,

You can check your network and sharing center.

Network and Sharing Center -> Advanced sharing settings ->Home or Work Profile Change

Thanks good work!

Handling errors in Promise.all

Promise.all is all or nothing. It resolves once all promises in the array resolve, or reject as soon as one of them rejects. In other words, it either resolves with an array of all resolved values, or rejects with a single error.

Some libraries have something called Promise.when, which I understand would instead wait for all promises in the array to either resolve or reject, but I'm not familiar with it, and it's not in ES6.

Your code

I agree with others here that your fix should work. It should resolve with an array that may contain a mix of successful values and errors objects. It's unusual to pass error objects in the success-path but assuming your code is expecting them, I see no problem with it.

The only reason I can think of why it would "not resolve" is that it's failing in code you're not showing us and the reason you're not seeing any error message about this is because this promise chain is not terminated with a final catch (as far as what you're showing us anyway).

I've taken the liberty of factoring out the "existing chain" from your example and terminating the chain with a catch. This may not be right for you, but for people reading this, it's important to always either return or terminate chains, or potential errors, even coding errors, will get hidden (which is what I suspect happened here):

Promise.all(state.routes.map(function(route) {
  return route.handler.promiseHandler().catch(function(err) {
    return err;
  });
}))
.then(function(arrayOfValuesOrErrors) {
  // handling of my array containing values and/or errors. 
})
.catch(function(err) {
  console.log(err.message); // some coding error in handling happened
});

Prevent flex items from overflowing a container

One easy solution is to use overflow values other than visible to make the text flex basis width reset as expected.

  1. Here with value auto the text wraps as expected and the article content does not overflow main container.

  2. Also, the article flex value must either have a auto basis AND be able to shrink, OR, only grow AND explicit 0 basis

_x000D_
_x000D_
main, aside, article {_x000D_
  margin: 10px;_x000D_
  border: solid 1px #000;_x000D_
  border-bottom: 0;_x000D_
  height: 50px;_x000D_
  overflow: auto; /* 1. overflow not `visible` */_x000D_
}_x000D_
main {_x000D_
  display: flex;_x000D_
}_x000D_
aside {_x000D_
  flex: 0 0 200px;_x000D_
}_x000D_
article {_x000D_
  flex: 1 1 auto; /* 2. Allow auto width content to shrink */_x000D_
  /* flex: 1 0 0; /* Or, explicit 0 width basis that grows */_x000D_
}
_x000D_
<main>_x000D_
  <aside>x x x x x x x x x x x x x x x x x x x x x x x x x x x x x x x x x x x x x x x x x x x x x x x x x x x x x x x x x x x x x x x x x x x x x x x x x x x x x x x x x x x x x x x x x x x x x x x </aside>_x000D_
  <article>don't let flex item overflow container.... y y y y y y y y y y y y y y y y y y y y y y y y y y y y y y y y y y y y y y y y y y y y y y y y y y y y y y y y y y y y y y y y y y y y y y y y y y y y y y y y y y y y y y y y y y y y y y y y y y y y y y y y y y y y y y y y y y y y y y y y y y y y y y y y y y y y y y y y y y y y y y y y y y y y y y y y y y y y y y y y y y y y y y y y y y y y y y y y y y y y y y y y y y y y y y y y y y y y y y y y y y y y y y y y y y y y y y y y y y y y y y y y y y y y y y y y y y y y y y y y y y y y y y y y y y y y y y y y y y y y y y y y y y y y y y y y y y y y y y y y y y y y y y y y y y y y y y y y y y y y y y y y y y y y y y y y y y y y y y y y y y y y y y y y y y y y y y y y </article>_x000D_
</main>
_x000D_
_x000D_
_x000D_

Change a web.config programmatically with C# (.NET)

Since web.config file is xml file you can open web.config using xmldocument class. Get the node from that xml file that you want to update and then save xml file.

here is URL that explains in more detail how you can update web.config file programmatically.

http://patelshailesh.com/index.php/update-web-config-programmatically

Note: if you make any changes to web.config, ASP.NET detects that changes and it will reload your application(recycle application pool) and effect of that is data kept in Session, Application, and Cache will be lost (assuming session state is InProc and not using a state server or database).

Model Binding to a List MVC 4

A clean solution could be create a generic class to handle the list, so you don't need to create a different class each time you need it.

public class ListModel<T>
{
    public List<T> Items { get; set; }

    public ListModel(List<T> list) {
        Items = list;
    }
}

and when you return the View you just need to simply do:

List<customClass> ListOfCustomClass = new List<customClass>();
//Do as needed...
return View(new ListModel<customClass>(ListOfCustomClass));

then define the list in the model:

@model ListModel<customClass>

and ready to go:

@foreach(var element in Model.Items) {
  //do as needed...
}

Find intersection of two nested lists?

The & operator takes the intersection of two sets.

{1, 2, 3} & {2, 3, 4}
Out[1]: {2, 3}

Include CSS,javascript file in Yii Framework

You can do so by adding

Yii::app()->clientScript->registerScriptFile(Yii::app()->baseUrl.'/path/to/your/script');

Selecting/excluding sets of columns in pandas

Here's how to create a copy of a DataFrame excluding a list of columns:

df = pd.DataFrame(np.random.randn(100, 4), columns=list('ABCD'))
df2 = df.drop(['B', 'D'], axis=1)

But be careful! You mention views in your question, suggesting that if you changed df, you'd want df2 to change too. (Like a view would in a database.)

This method doesn't achieve that:

>>> df.loc[0, 'A'] = 999 # Change the first value in df
>>> df.head(1)
     A         B         C         D
0  999 -0.742688 -1.980673 -0.920133
>>> df2.head(1) # df2 is unchanged. It's not a view, it's a copy!
          A         C
0  0.251262 -1.980673

Note also that this is also true of @piggybox's method. (Although that method is nice and slick and Pythonic. I'm not doing it down!!)

For more on views vs. copies see this SO answer and this part of the Pandas docs which that answer refers to.

Determine what attributes were changed in Rails after_save callback?

The "selected" answer didn't work for me. I'm using rails 3.1 with CouchRest::Model (based on Active Model). The _changed? methods don't return true for changed attributes in the after_update hook, only in the before_update hook. I was able to get it to work using the (new?) around_update hook:

class SomeModel < ActiveRecord::Base
  around_update :send_notification_after_change

  def send_notification_after_change
    should_send_it = self.published_changed? && self.published == true

    yield

    Notification.send(...) if should_send_it
  end

end

What is the { get; set; } syntax in C#?

That is an Auto-Implemented Property. It's basically a shorthand way of creating properties for a class in C#, without having to define private variables for them. They are normally used when no extra logic is required when getting or setting the value of a variable.

You can read more on MSDN's Auto-Implemented Properties Programming Guide.

Recommended way to embed PDF in HTML?

Have a look for this code- To embed the PDF in HTML

<!-- Embed PDF File -->
<object data="YourFile.pdf" type="application/x-pdf" title="SamplePdf" width="500" height="720">
    <a href="YourFile.pdf">shree</a> 
</object>

How to manually set an authenticated user in Spring Security / SpringMVC

I was trying to test an extjs application and after sucessfully setting a testingAuthenticationToken this suddenly stopped working with no obvious cause.

I couldn't get the above answers to work so my solution was to skip out this bit of spring in the test environment. I introduced a seam around spring like this:

public class SpringUserAccessor implements UserAccessor
{
    @Override
    public User getUser()
    {
        SecurityContext context = SecurityContextHolder.getContext();
        Authentication authentication = context.getAuthentication();
        return (User) authentication.getPrincipal();
    }
}

User is a custom type here.

I'm then wrapping it in a class which just has an option for the test code to switch spring out.

public class CurrentUserAccessor
{
    private static UserAccessor _accessor;

    public CurrentUserAccessor()
    {
        _accessor = new SpringUserAccessor();
    }

    public User getUser()
    {
        return _accessor.getUser();
    }

    public static void UseTestingAccessor(User user)
    {
        _accessor = new TestUserAccessor(user);
    }
}

The test version just looks like this:

public class TestUserAccessor implements UserAccessor
{
    private static User _user;

    public TestUserAccessor(User user)
    {
        _user = user;
    }

    @Override
    public User getUser()
    {
        return _user;
    }
}

In the calling code I'm still using a proper user loaded from the database:

    User user = (User) _userService.loadUserByUsername(username);
    CurrentUserAccessor.UseTestingAccessor(user);

Obviously this wont be suitable if you actually need to use the security but I'm running with a no-security setup for the testing deployment. I thought someone else might run into a similar situation. This is a pattern I've used for mocking out static dependencies before. The other alternative is you can maintain the staticness of the wrapper class but I prefer this one as the dependencies of the code are more explicit since you have to pass CurrentUserAccessor into classes where it is required.

Which type of folder structure should be used with Angular 2?

I suggest the following structure, which might violate some existing conventions.

I was striving to reduce name redundancy in the path, and trying to keep naming short in general.

So there is no/app/components/home/home.component.ts|html|css.

Instead it looks like this:

|-- app
    |-- users
        |-- list.ts|html|css
        |-- form.ts|html|css
    |-- cars
        |-- list.ts|html|css
        |-- form.ts|html|css
        |-- configurator.ts|html|css
    |-- app.component.ts|html|css
    |-- app.module.ts
    |-- user.service.ts
    |-- car.service.ts
|-- index.html
|-- main.ts
|-- style.css

How to select first child with jQuery?

Try with: $('.onediv').eq(0)

demo jsBin

From the demo: Other examples of selectors and methods targeting the first LI unside an UL:

.eq() Method: $('li').eq(0)
:eq() selector: $('li:eq(0)')
.first() Method $('li').first()
:first selector: $('li:first')
:first-child selector: $('li:first-child')
:lt() selector:$('li:lt(1)')
:nth-child() selector:$('li:nth-child(1)')

jQ + JS:

Array.slice() Method: $('li').slice(0,1)

you can also use [i] to get the JS HTMLelement index out of the jQuery el. (array) collection like eg:

$('li')[0]

now that you have the JS element representation you have to use JS native methods eg:

$('li')[0].className = 'active'; // Adds class "active" to the first LI in the DOM

or you can (don't - it's bad design) wrap it back into a jQuery object

$( $('li')[0] ).addClass('active'); // Don't. Use .eq() instead

How can I use a C++ library from node.js?

Becareful with swig and C++: http://www.swig.org/Doc1.3/SWIG.html#SWIG_nn8

Running SWIG on C++ source files (what would appear in a .C or .cxx file) is not recommended. Even though SWIG can parse C++ class declarations, it ignores declarations that are decoupled from their original class definition (the declarations are parsed, but a lot of warning messages may be generated). For example:

/* Not supported by SWIG */
int foo::bar(int) {
    ... whatever ...
}

It's rarely to have a C++ class limited to only one .h file.

Also, the versions of swig supporting JavaScript is swig-3.0.1 or later.

How to prevent colliders from passing through each other?

I have a pinball prototype that also gave me much trouble in the same areas. These are all the steps I've taken to almost (but not yet entirely) solve these problems:

For fast moving objects:

  • Set the rigidbody's Interpolate to 'Interpolate' (this does not affect the actual physics simulation, but updates the rendering of the object properly - use this only on important objects from a rendering point of view, like the player, or a pinball, but not for projectiles)

  • Set Collision Detection to Continuous Dynamic

  • Attach the script DontGoThroughThings (https://www.auto.tuwien.ac.at/wordpress/?p=260) to your object. This script cleverly uses the Raycasting solution I posted in my other answer to pull back offending objects to before the collision points.

In Edit -> Project Settings -> Physics:

  • Set Min Penetration for Penalty to a very low value. I've set mine to 0.001

  • Set Solver Iteration Count to a higher value. I've set mine to 50, but you can probably do ok with much less.

All that is going to have a penalty in performace, but that's unavoidable. The defaults values are soft on performance but are not really intented for proper simulation of small and fast-moving objects.

Validate date in dd/mm/yyyy format using JQuery Validate

I encountered a similar problem in my project. After struggling a lot, I found this solution:

if ($.datepicker.parseDate("dd/mm/yy","17/06/2015") > $.datepicker.parseDate("dd/mm/yy","20/06/2015"))
    // do something

You DO NOT NEED plugins like jQuery Validate or Moment.js for this issue. Hope this solution helps.

How to sort Counter by value? - python

A rather nice addition to @MartijnPieters answer is to get back a dictionary sorted by occurrence since Collections.most_common only returns a tuple. I often couple this with a json output for handy log files:

from collections import Counter, OrderedDict

x = Counter({'a':5, 'b':3, 'c':7})
y = OrderedDict(x.most_common())

With the output:

OrderedDict([('c', 7), ('a', 5), ('b', 3)])
{
  "c": 7, 
  "a": 5, 
  "b": 3
}

Converting a double to an int in Javascript without rounding

As @Quentin and @Pointy pointed out in their comments, it's not a good idea to use parseInt() because it is designed to convert a string to an integer. When you pass a decimal number to it, it first converts the number to a string, then casts it to an integer. I suggest you use Math.trunc(), Math.floor(), ~~num, ~~v , num | 0, num << 0, or num >> 0 depending on your needs. This performance test demonstrates the difference in parseInt() and Math.floor() performance. Also, this post explains the difference between the proposed methods.

Read line by line in bash script

while read CMD; do
    echo $CMD
done  << EOF
data line 1
data line 2
..
EOF

Simple JavaScript Checkbox Validation

var confirm=document.getElementById("confirm").value;
         if((confirm.checked==false)
         {
         alert("plz check the checkbox field");
         document.getElementbyId("confirm").focus();
         return false;
         }

How to suppress Update Links warning?

I've found a temporary solution that will at least let me process this job. I wrote a short AutoIt script that waits for the "Update Links" window to appear, then clicks the "Don't Update" button. Code is as follows:

while 1
if winexists("Microsoft Excel","This workbook contains links to other data sources.") Then
   controlclick("Microsoft Excel","This workbook contains links to other data sources.",2)
EndIf
WEnd

So far this seems to be working. I'd really like to find a solution that's entirely VBA, however, so that I can make this a standalone application.

Synchronous Requests in Node.js

The short answer is: don't. If you want code that reads linearly, use a library like seq. But just don't expect synchronous. You really can't. And that's a good thing.

There's little or nothing that can't be put in a callback. If they depend on common variables, create a closure to contain them. What's the actual task at hand?

You'd want to have a counter, and only call the callback when the data is there:

var waiting = 2;
request( {url: base + u_ext}, function( err, res, body ) {
    var split1 = body.split("\n");
    var split2 = split1[1].split(", ");
    ucomp = split2[1];
    if(--waiting == 0) callback();
});

request( {url: base + v_ext}, function( err, res, body ) {
    var split1 = body.split("\n");
    var split2 = split1[1].split(", ");
    vcomp = split2[1];
    if(--waiting == 0) callback();
});

function callback() {
    // do math here.
}

Update 2018: node.js supports async/await keywords in recent editions, and with libraries that represent asynchronous processes as promises, you can await them. You get linear, sequential flow through your program, and other work can progress while you await. It's pretty well built and worth a try.

How to put data containing double-quotes in string variable?

You can escape (this is how this principle is called) the double quotes by prefixing them with another double quote. You can put them in a string as follows:

Dim MyVar as string = "some text ""hello"" "

This will give the MyVar variable a value of some text "hello".

fast way to copy formatting in excel

Does:

Set Sheets("Output").Range("$A$1:$A$500") =  Sheets(sheet_).Range("$A$1:$A$500")

...work? (I don't have Excel in front of me, so can't test.)

What is the best (idiomatic) way to check the type of a Python variable?

That should work - so no, there is nothing wrong with your code. However, it could also be done with a dict:

{type(str()): do_something_with_a_string,
 type(dict()): do_something_with_a_dict}.get(type(x), errorhandler)()

A bit more concise and pythonic wouldn't you say?


Edit.. Heeding Avisser's advice, the code also works like this, and looks nicer:

{str: do_something_with_a_string,
 dict: do_something_with_a_dict}.get(type(x), errorhandler)()

How to get the parents of a Python class?

The FASTEST way, to see all parents, and IN ORDER, just use the built in __mro__

i.e. repr(YOUR_CLASS.__mro__)


>>>
>>>
>>> import getpass
>>> getpass.GetPassWarning.__mro__

outputs, IN ORDER


(<class 'getpass.GetPassWarning'>, <type 'exceptions.UserWarning'>,
<type 'exceptions.Warning'>, <type 'exceptions.Exception'>, 
<type 'exceptions.BaseException'>, <type 'object'>)
>>>

There you have it. The "best" answer right now, has 182 votes (as I am typing this) but this is SO much simpler than some convoluted for loop, looking into bases one class at a time, not to mention when a class extends TWO or more parent classes. Importing and using inspect just clouds the scope unnecessarily. It honestly is a shame people don't know to just use the built-ins

I Hope this Helps!

Show DialogFragment with animation growing from a point

Add this code on values anim

 <scale
    android:duration="@android:integer/config_longAnimTime"
    android:fromXScale="0.2"
    android:fromYScale="0.2"
    android:toXScale="1.0"
    android:toYScale="1.0"
    android:pivotX="50%"
    android:pivotY="50%"/>
<alpha
    android:fromAlpha="0.1"
    android:toAlpha="1.0"
    android:duration="@android:integer/config_longAnimTime"
    android:interpolator="@android:anim/accelerate_decelerate_interpolator"/>

call on styles.xml

<style name="DialogScale">
    <item name="android:windowEnterAnimation">@anim/scale_in</item>
    <item name="android:windowExitAnimation">@anim/scale_out</item>
</style>

On java code: set Onclick

public void onClick(View v) {
        fab_onclick(R.style.DialogScale, "Scale" ,(Activity) context,getWindow().getDecorView().getRootView());
      //  Dialogs.fab_onclick(R.style.DialogScale, "Scale");

    }

setup on method:

alertDialog.getWindow().getAttributes().windowAnimations = type;

Defined Edges With CSS3 Filter Blur

In the many situations where the IMG can be made position:absolute, you can use clip to hide the blurred edges--and the outer DIV is unnecessary.

img {
    filter: blur(5px);
        -webkit-filter: blur(5px);
        -moz-filter: blur(5px);
        -o-filter: blur(5px);
        -ms-filter: blur(5px);
    position: absolute;
    clip: rect(5px,295px,295px;5px);
}

How to overcome "'aclocal-1.15' is missing on your system" warning?

2017 - High Sierra

It is really hard to get autoconf 1.15 working on Mac. We hired an expert to get it working. Everything worked beautifully.

Later I happened to upgrade a Mac to High Sierra.

The Docker pipeline stopped working!

Even though autoconf 1.15 is working fine on the Mac.

How to fix,

Short answer, I simply trashed the local repo, and checked out the repo again.

This suggestion is noted in the mix on this QA page and elsewhere.

It then worked fine!

It likely has something to do with the aclocal.m4 and similar files. (But who knows really). I endlessly massaged those files ... but nothing.

For some unknown reason if you just scratch your repo and get the repo again: everything works!

I tried for hours every combo of touching/deleting etc etc the files in question, but no. Just check out the repo from scratch!

How to execute raw queries with Laravel 5.1?

I found the solution in this topic and I code this:

$cards = DB::select("SELECT
        cards.id_card,
        cards.hash_card,
        cards.`table`,
        users.name,
        0 as total,
        cards.card_status,
        cards.created_at as last_update
    FROM cards
    LEFT JOIN users
    ON users.id_user = cards.id_user
    WHERE hash_card NOT IN ( SELECT orders.hash_card FROM orders )
    UNION
    SELECT
        cards.id_card,
        orders.hash_card,
        cards.`table`,
        users.name,
        sum(orders.quantity*orders.product_price) as total, 
        cards.card_status, 
        max(orders.created_at) last_update 
    FROM menu.orders
    LEFT JOIN cards
    ON cards.hash_card = orders.hash_card
    LEFT JOIN users
    ON users.id_user = cards.id_user
    GROUP BY hash_card
    ORDER BY id_card ASC");

appcompat-v7:21.0.0': No resource found that matches the given name: attr 'android:actionModeShareDrawable'

Make sure you clean your project in android studio (or eclipse),

It should solve your issues

How to add additional fields to form before submit?

May be useful for some:

(a function that allow you to add the data to the form using an object, with override for existing inputs, if there is) [pure js]

(form is a dom el, and not a jquery object [jqryobj.get(0) if you need])

function addDataToForm(form, data) {
    if(typeof form === 'string') {
        if(form[0] === '#') form = form.slice(1);
        form = document.getElementById(form);
    }

    var keys = Object.keys(data);
    var name;
    var value;
    var input;

    for (var i = 0; i < keys.length; i++) {
        name = keys[i];
        // removing the inputs with the name if already exists [overide]
        // console.log(form);
        Array.prototype.forEach.call(form.elements, function (inpt) {
             if(inpt.name === name) {
                 inpt.parentNode.removeChild(inpt);
             }
        });

        value = data[name];
        input = document.createElement('input');
        input.setAttribute('name', name);
        input.setAttribute('value', value);
        input.setAttribute('type', 'hidden');

        form.appendChild(input);
    }

    return form;
}

Use :

addDataToForm(form, {
    'uri': window.location.href,
     'kpi_val': 150,
     //...
});

you can use it like that too

var form = addDataToForm('myFormId', {
    'uri': window.location.href,
     'kpi_val': 150,
     //...
});

you can add # if you like too ("#myformid").

How to attach a process in gdb

Try one of these:

gdb -p 12271
gdb /path/to/exe 12271

gdb /path/to/exe
(gdb) attach 12271

.Net HttpWebRequest.GetResponse() raises exception when http status code 400 (bad request) is returned

I know this has already been answered a long time ago, but I made an extension method to hopefully help other people that come to this question.

Code:

public static class WebRequestExtensions
{
    public static WebResponse GetResponseWithoutException(this WebRequest request)
    {
        if (request == null)
        {
            throw new ArgumentNullException("request");
        }

        try
        {
            return request.GetResponse();
        }
        catch (WebException e)
        {
            if (e.Response == null)
            {
                throw;
            }

            return e.Response;
        }
    }
}

Usage:

var request = (HttpWebRequest)WebRequest.CreateHttp("http://invalidurl.com");

//... (initialize more fields)

using (var response = (HttpWebResponse)request.GetResponseWithoutException())
{
    Console.WriteLine("I got Http Status Code: {0}", response.StatusCode);
}

How to get previous month and year relative to today, using strtotime and date?

function getOnemonthBefore($date){
    $day = intval(date("t", strtotime("$date")));//get the last day of the month
    $month_date = date("y-m-d",strtotime("$date -$day days"));//get the day 1 month before
    return $month_date;
}

The resulting date is dependent to the number of days the input month is consist of. If input month is february (28 days), 28 days before february 5 is january 8. If input is may 17, 31 days before is april 16. Likewise, if input is may 31, resulting date will be april 30.

NOTE: the input takes complete date ('y-m-d') and outputs ('y-m-d') you can modify this code to suit your needs.

how do I set height of container DIV to 100% of window height?

html {
  min-height: 100%;
}

body {
  min-height: 100vh;
}

The html height (%) will take care of the height of the documents that's height is more than a 100% of the screen view while the body view height (vh) will take care of the document's height that is less than the height of the screen view.

What's a "static method" in C#?

The static keyword, when applied to a class, tells the compiler to create a single instance of that class. It is not then possible to 'new' one or more instance of the class. All methods in a static class must themselves be declared static.

It is possible, And often desirable, to have static methods of a non-static class. For example a factory method when creates an instance of another class is often declared static as this means that a particular instance of the class containing the factor method is not required.

For a good explanation of how, when and where see MSDN

Reloading submodules in IPython

For some reason, neither %autoreload, nor dreload seem to work for the situation when you import code from one notebook to another. Only plain Python reload works:

reload(module)

Based on [1].

Jquery ajax call click event submit button

You did not add # before id of the button. You do not have right selector in your jquery code. So jquery is never execute in your button click. its submitted your form directly not passing any ajax request.

See documentation: http://api.jquery.com/category/selectors/
its your friend.

Try this:

It seems that id: $("#Shareitem").val() is wrong if you want to pass the value of

<input type="hidden" name="id" value="" id="id">

you need to change this line:

id: $("#Shareitem").val()

by

id: $("#id").val()

All together:

 <script src="http://code.jquery.com/jquery-1.11.0.min.js"></script>
    <script>
    $(document).ready(function(){
      $("#Shareitem").click(function(e){
          e.preventDefault();
        $.ajax({type: "POST",
                url: "/imball-reagens/public/shareitem",
                data: { id: $("#Shareitem").val(), access_token: $("#access_token").val() },
                success:function(result){
          $("#sharelink").html(result);
        }});
      });
    });
    </script>

JavaScript to scroll long page to DIV

This worked for me

document.getElementById('divElem').scrollIntoView();

Serializing an object as UTF-8 XML in .NET

I found this blog post which explains the problem very well, and defines a few different solutions:

(dead link removed)

I've settled for the idea that the best way to do it is to completely omit the XML declaration when in memory. It actually is UTF-16 at that point anyway, but the XML declaration doesn't seem meaningful until it has been written to a file with a particular encoding; and even then the declaration is not required. It doesn't seem to break deserialization, at least.

As @Jon Hanna mentions, this can be done with an XmlWriter created like this:

XmlWriter writer = XmlWriter.Create (output, new XmlWriterSettings() { OmitXmlDeclaration = true });

Stretch Image to Fit 100% of Div Height and Width

will the height attribute stretch the image beyond its native resolution? If I have a image with a height of say 420 pixels, I can't get css to stretch the image beyond the native resolution to fill the height of the viewport.

I am getting pretty close results with:

 .rightdiv img {
        max-width: 25vw;
        min-height: 100vh;
    }

the 100vh is getting pretty close, with just a few pixels left over at the bottom for some reason.

Declaring functions in JSP?

You need to enclose that in <%! %> as follows:

<%!

public String getQuarter(int i){
String quarter;
switch(i){
        case 1: quarter = "Winter";
        break;

        case 2: quarter = "Spring";
        break;

        case 3: quarter = "Summer I";
        break;

        case 4: quarter = "Summer II";
        break;

        case 5: quarter = "Fall";
        break;

        default: quarter = "ERROR";
}

return quarter;
}

%>

You can then invoke the function within scriptlets or expressions:

<%
     out.print(getQuarter(4));
%>

or

<%= getQuarter(17) %>

Disable form autofill in Chrome without disabling autocomplete

I stumbled upon the weird chrome autofill behaviour today. It happened to enable on fields called: "embed" and "postpassword" (filling there login and password) with no apparent reason. Those fields had already autocomplete set to off.

None of the described methods seemed to work. None of the methods from the another answer worked as well. I came upon my own idea basing on Steele's answer (it might have actually worked, but I require the fixed post data format in my application):

Before the real password, add those two dummy fields:

<input type='text' style='display: none'>
<input type='password' style='display: none'>

Only this one finally disabled autofill altogether for my form.

It's a shame, that disabling such a basic behavior is that hard and hacky.

jQuery UI Accordion Expand/Collapse All

Yes, it is possible. Put all div in separate accordion class as follows:

<script type="text/javascript" src="jquery.js"></script>
<script type="text/javascript" src="jquery-ui.js"></script>

<script type="text/javascript">

        $(function () {
            $("input[type=submit], button")
        .button()
        .click(function (event) {
            event.preventDefault();
        });
            $("#tabs").tabs();
            $(".accordion").accordion({
                heightStyle: "content",

                collapsible: true,
                active: 0



            });
        });

function expandAll()
{
  $(".accordion").accordion({
                heightStyle: "content",

                collapsible: true,
                active: 0

            });

            return false;   
}

function collapseAll()
{
  $(".accordion").accordion({
                heightStyle: "content",

                collapsible: true,
                active: false



            });
            return false;
}
</script>



<div class="accordion">
  <h3>Toggle 1</h3>
  <div >
    <p>text1.</p>
  </div>
</div>
<div class="accordion">
  <h3>Toggle 2</h3>
  <div >
    <p>text2.</p>
  </div>
</div>
<div class="accordion">
  <h3>Toggle 3</h3>
  <div >
    <p>text3.</p>
  </div>
</div>

npm install error - unable to get local issuer certificate

In case you use yarn:

yarn config set strict-ssl false

gdb fails with "Unable to find Mach task port for process-id" error

This link had the clearest and most detailed step-by-step to make this error disappear for me.

In my case I had to have the key as a "System" key otherwise it did not work (which not every url mentions).

Also killing taskgated is a viable (and quicker) alternative to having to restart.

I also uninstalled MacPorts before I started this process and uninstalled the current gdb using brew uninstall gdb.

Decompile .smali files on an APK

dex2jar helps to decompile your apk but not 100%. You will have some problems with .smali files. Dex2jar cannot convert it to java. I know one application that can decompile your apk source files and no problems with .smali files. Here is a link http://www.hensence.com/en/smali2java/

R: numeric 'envir' arg not of length one in predict()

There are several problems here:

  1. The newdata argument of predict() needs a predictor variable. You should thus pass it values for Coupon, instead of Total, which is the response variable in your model.

  2. The predictor variable needs to be passed in as a named column in a data frame, so that predict() knows what the numbers its been handed represent. (The need for this becomes clear when you consider more complicated models, having more than one predictor variable).

  3. For this to work, your original call should pass df in through the data argument, rather than using it directly in your formula. (This way, the name of the column in newdata will be able to match the name on the RHS of the formula).

With those changes incorporated, this will work:

model <- lm(Total ~ Coupon, data=df)
new <- data.frame(Coupon = df$Coupon)
predict(model, newdata = new, interval="confidence")

How to get MAC address of your machine using a C program?

  1. On Linux, use the service of "Network Manager" over the DBus.

  2. There is also good'ol shell program which can be invoke and the result grabbed (use an exec function under C):

$ /sbin/ifconfig | grep HWaddr

React-Router External link

If you are using server side rending, you can use StaticRouter. With your context as props and then adding <Redirect path="/somewhere" /> component in your app. The idea is everytime react-router matches a redirect component it will add something into the context you passed into the static router to let you know your path matches a redirect component. now that you know you hit a redirect you just need to check if thats the redirect you are looking for. then just redirect through the server. ctx.redirect('https://example/com').

Append value to empty vector in R?

Appending to an object in a for loop causes the entire object to be copied on every iteration, which causes a lot of people to say "R is slow", or "R loops should be avoided".

As BrodieG mentioned in the comments: it is much better to pre-allocate a vector of the desired length, then set the element values in the loop.

Here are several ways to append values to a vector. All of them are discouraged.

Appending to a vector in a loop

# one way
for (i in 1:length(values))
  vector[i] <- values[i]
# another way
for (i in 1:length(values))
  vector <- c(vector, values[i])
# yet another way?!?
for (v in values)
  vector <- c(vector, v)
# ... more ways

help("append") would have answered your question and saved the time it took you to write this question (but would have caused you to develop bad habits). ;-)

Note that vector <- c() isn't an empty vector; it's NULL. If you want an empty character vector, use vector <- character().

Pre-allocate the vector before looping

If you absolutely must use a for loop, you should pre-allocate the entire vector before the loop. This will be much faster than appending for larger vectors.

set.seed(21)
values <- sample(letters, 1e4, TRUE)
vector <- character(0)
# slow
system.time( for (i in 1:length(values)) vector[i] <- values[i] )
#   user  system elapsed 
#  0.340   0.000   0.343 
vector <- character(length(values))
# fast(er)
system.time( for (i in 1:length(values)) vector[i] <- values[i] )
#   user  system elapsed 
#  0.024   0.000   0.023 

Increment a database field by 1

I not expert in MySQL but you probably should look on triggers e.g. BEFORE INSERT. In the trigger you can run select query on your original table and if it found something just update the row 'logins' instead of inserting new values. But all this depends on version of MySQL you running.

Left Join without duplicate rows from left table

Try an OUTER APPLY

SELECT 
    C.Content_ID,
    C.Content_Title,
    C.Content_DatePublished,
    M.Media_Id
FROM 
    tbl_Contents C
    OUTER APPLY
    (
        SELECT TOP 1 *
        FROM tbl_Media M 
        WHERE M.Content_Id = C.Content_Id 
    ) m
ORDER BY 
    C.Content_DatePublished ASC

Alternatively, you could GROUP BY the results

SELECT 
    C.Content_ID,
    C.Content_Title,
    C.Content_DatePublished,
    M.Media_Id
FROM 
    tbl_Contents C
    LEFT OUTER JOIN tbl_Media M ON M.Content_Id = C.Content_Id 
GROUP BY
    C.Content_ID,
    C.Content_Title,
    C.Content_DatePublished,
    M.Media_Id
ORDER BY
    C.Content_DatePublished ASC

The OUTER APPLY selects a single row (or none) that matches each row from the left table.

The GROUP BY performs the entire join, but then collapses the final result rows on the provided columns.

Space between two divs

You can try something like the following:

h1{
   margin-bottom:<x>px;
}
div{
   margin-bottom:<y>px;
}
div:last-of-type{
   margin-bottom:0;
}

or instead of the first h1 rule:

div:first-of-type{
   margin-top:<x>px;
}

or even better use the adjacent sibling selector. With the following selector, you could cover your case in one rule:

div + div{
   margin-bottom:<y>px;
}

Respectively, h1 + div would control the first div after your header, giving you additional styling options.

Create File If File Does Not Exist

For Example

    string rootPath = Path.GetPathRoot(Environment.GetFolderPath(Environment.SpecialFolder.System));
        rootPath += "MTN";
        if (!(File.Exists(rootPath)))
        {
            File.CreateText(rootPath);
        }

How to Get the Query Executed in Laravel 5? DB::getQueryLog() Returning Empty Array

(Laravel 5.2) I find the simplest way is just to add one code line to monitor the sql queries:

\DB::listen(function($sql) {var_dump($sql); });

Printing prime numbers from 1 through 100

If j is equal to sqrt(i) it might also be a valid factor, not only if it's smaller.

To iterate up to and including sqrt(i) in your inner loop, you could write:

for (int j=2; j*j<=i; j++)

(Compared to using sqrt(i) this has the advantage to not need conversion to floating point numbers.)

increase the java heap size permanently?

Apparently, _JAVA_OPTIONS works on Linux, too:

$ export _JAVA_OPTIONS="-Xmx1g"
$ java -jar jconsole.jar &
Picked up _JAVA_OPTIONS: -Xmx1g

How to delete files recursively from an S3 bucket

You might also consider using Amazon S3 Lifecycle to create an expiration for files with the prefix foo/bar1.

Open the S3 browser console and click a bucket. Then click Properties and then LifeCycle.

Create an expiration rule for all files with the prefix foo/bar1 and set the date to 1 day since file was created.

Save and all matching files will be gone within 24 hours.

Just don't forget to remove the rule after you're done!

No API calls, no third party libraries, apps or scripts.

I just deleted several million files this way.

A screenshot showing the Lifecycle Rule window (note in this shot the Prefix has been left blank, affecting all keys in the bucket):

enter image description here

What is the meaning of the word logits in TensorFlow?

Here is a concise answer for future readers. Tensorflow's logit is defined as the output of a neuron without applying activation function:

logit = w*x + b,

x: input, w: weight, b: bias. That's it.


The following is irrelevant to this question.

For historical lectures, read other answers. Hats off to Tensorflow's "creatively" confusing naming convention. In PyTorch, there is only one CrossEntropyLoss and it accepts un-activated outputs. Convolutions, matrix multiplications and activations are same level operations. The design is much more modular and less confusing. This is one of the reasons why I switched from Tensorflow to PyTorch.

Error in <my code> : object of type 'closure' is not subsettable

In general this error message means that you have tried to use indexing on a function. You can reproduce this error message with, for example

mean[1]
## Error in mean[1] : object of type 'closure' is not subsettable
mean[[1]]
## Error in mean[[1]] : object of type 'closure' is not subsettable
mean$a
## Error in mean$a : object of type 'closure' is not subsettable

The closure mentioned in the error message is (loosely) the function and the environment that stores the variables when the function is called.


In this specific case, as Joshua mentioned, you are trying to access the url function as a variable. If you define a variable named url, then the error goes away.

As a matter of good practise, you should usually avoid naming variables after base-R functions. (Calling variables data is a common source of this error.)


There are several related errors for trying to subset operators or keywords.

`+`[1]
## Error in `+`[1] : object of type 'builtin' is not subsettable
`if`[1]
## Error in `if`[1] : object of type 'special' is not subsettable

If you're running into this problem in shiny, the most likely cause is that you're trying to work with a reactive expression without calling it as a function using parentheses.

library(shiny)
reactive_df <- reactive({
    data.frame(col1 = c(1,2,3),
               col2 = c(4,5,6))
})

While we often work with reactive expressions in shiny as if they were data frames, they are actually functions that return data frames (or other objects).

isolate({
    print(reactive_df())
    print(reactive_df()$col1)
})
  col1 col2
1    1    4
2    2    5
3    3    6
[1] 1 2 3

But if we try to subset it without parentheses, then we're actually trying to index a function, and we get an error:

isolate(
    reactive_df$col1
)
Error in reactive_df$col1 : object of type 'closure' is not subsettable

SQL Server database backup restore on lower version

You can try this.

  1. Create a Database onto SQL Server 2008.
  2. Using Import Data feature import data from SQL Server R2 (or any higher version).
  3. use "RedGate SQLCompare" to synchronize script.

Angularjs - ng-cloak/ng-show elements blink

I'm using ng-show in a directive to show and hide popups.

<div class="..." ng-show="showPopup">

None of the above worked for me, and using ng-if instead of ng-show would be an overkill. That would imply removing and adding the whole popup content into the DOM at every single click. Instead I added an ng-if into the same element to make sure it doesn't show at the document load:

<div class="..." ng-show="showPopup" ng-if="popupReady">

Afterwards I added the initialization into the controller responsible of this directive with a timeout:

$timeout(function () {
    $scope.popupReady = true;
});

This way I eliminated the flickering issue and avoided the costly operation of DOM insertion at every single click. This came at an expense of using two scope variables for the same purpose instead of one, but so far this is definitely the best option.

phpmysql error - #1273 - #1273 - Unknown collation: 'utf8mb4_general_ci'

There are two steps to fix this.

First edit phpMyAdmin/libraries/DatabaseInterface.class.php

Change:

    if (PMA_MYSQL_INT_VERSION >  50503) {
        $default_charset = 'utf8mb4';
        $default_collation = 'utf8mb4_general_ci';
    } else {
        $default_charset = 'utf8';
        $default_collation = 'utf8_general_ci';
    }

To:

    //if (PMA_MYSQL_INT_VERSION >  50503) {
    //    $default_charset = 'utf8mb4';
    //    $default_collation = 'utf8mb4_general_ci';
    //} else {
        $default_charset = 'utf8';
        $default_collation = 'utf8_general_ci';
    //}

Then delete this cookie from your browser "pma_collation_connection".
Or delete all Cookies.

Then restart your phpMyAdmin.

(It would be nice if phpMyAdmin allowed you to set the charset and collation per server in the config.inc.php)

The difference between "require(x)" and "import x"

Not an answer here and more like a comment, sorry but I can't comment.

In node V10, you can use the flag --experimental-modules to tell Nodejs you want to use import. But your entry script should end with .mjs.

Note this is still an experimental thing and should not be used in production.

// main.mjs
import utils from './utils.js'
utils.print();
// utils.js
module.exports={
    print:function(){console.log('print called')}
}

Ref 1 - Nodejs Doc

Ref 2 - github issue

HashMaps and Null values?

Acording to your first code snipet seems ok, but I've got similar behavior caused by bad programing. Have you checked the "options" variable is not null before the put call?

I'm using Struts2 (2.3.3) webapp and use a HashMap for displaying results. When is executed (in a class initialized by an Action class) :

if(value != null) pdfMap.put("date",value.toString());
else pdfMap.put("date","");

Got this error:

Struts Problem Report

Struts has detected an unhandled exception:

Messages:   
File:   aoc/psisclient/samples/PDFValidation.java
Line number:    155
Stacktraces

java.lang.NullPointerException
    aoc.psisclient.samples.PDFValidation.getRevisionsDetail(PDFValidation.java:155)
    aoc.action.signature.PDFUpload.execute(PDFUpload.java:66)
    sun.reflect.NativeMethodAccessorImpl.invoke0(Native Method)
    sun.reflect.NativeMethodAccessorImpl.invoke(NativeMethodAccessorImpl.java:39)
    ...

Seems the NullPointerException points to the put method (Line number 155), but the problem was that de Map hasn't been initialized before. It compiled ok since the variable is out of the method that set the value.

Select last row in MySQL

Keep in mind that tables in relational databases are just sets of rows. And sets in mathematics are unordered collections. There is no first or last row; no previous row or next row.

You'll have to sort your set of unordered rows by some field first, and then you are free the iterate through the resultset in the order you defined.

Since you have an auto incrementing field, I assume you want that to be the sorting field. In that case, you may want to do the following:

SELECT    *
FROM      your_table
ORDER BY  your_auto_increment_field DESC
LIMIT     1;

See how we're first sorting the set of unordered rows by the your_auto_increment_field (or whatever you have it called) in descending order. Then we limit the resultset to just the first row with LIMIT 1.

Incrementing a variable inside a Bash loop

You're getting final 0 because your while loop is being executed in a sub (shell) process and any changes made there are not reflected in the current (parent) shell.

Correct script:

while read -r country _; do
  if [ "US" = "$country" ]; then
        ((USCOUNTER++))
        echo "US counter $USCOUNTER"
  fi
done < "$FILE"

get current page from url

Try this:

path.Substring(path.LastIndexOf("/");

How to wait for a JavaScript Promise to resolve before resuming function?

I'm wondering if there is any way to get a value from a Promise or wait (block/sleep) until it has resolved, similar to .NET's IAsyncResult.WaitHandle.WaitOne(). I know JavaScript is single-threaded, but I'm hoping that doesn't mean that a function can't yield.

The current generation of Javascript in browsers does not have a wait() or sleep() that allows other things to run. So, you simply can't do what you're asking. Instead, it has async operations that will do their thing and then call you when they're done (as you've been using promises for).

Part of this is because of Javascript's single threadedness. If the single thread is spinning, then no other Javascript can execute until that spinning thread is done. ES6 introduces yield and generators which will allow some cooperative tricks like that, but we're quite a ways from being able to use those in a wide swatch of installed browsers (they can be used in some server-side development where you control the JS engine that is being used).


Careful management of promise-based code can control the order of execution for many async operations.

I'm not sure I understand exactly what order you're trying to achieve in your code, but you could do something like this using your existing kickOff() function, and then attaching a .then() handler to it after calling it:

function kickOff() {
  return new Promise(function(resolve, reject) {
    $("#output").append("start");

    setTimeout(function() {
      resolve();
    }, 1000);
  }).then(function() {
    $("#output").append(" middle");
    return " end";
  });
}

kickOff().then(function(result) {
    // use the result here
    $("#output").append(result);
});

This will return output in a guaranteed order - like this:

start
middle
end

Update in 2018 (three years after this answer was written):

If you either transpile your code or run your code in an environment that supports ES7 features such as async and await, you can now use await to make your code "appear" to wait for the result of a promise. It is still developing with promises. It does still not block all of Javascript, but it does allow you to write sequential operations in a friendlier syntax.

Instead of the ES6 way of doing things:

someFunc().then(someFunc2).then(result => {
    // process result here
}).catch(err => {
    // process error here
});

You can do this:

// returns a promise
async function wrapperFunc() {
    try {
        let r1 = await someFunc();
        let r2 = await someFunc2(r1);
        // now process r2
        return someValue;     // this will be the resolved value of the returned promise
    } catch(e) {
        console.log(e);
        throw e;      // let caller know the promise was rejected with this reason
    }
}

wrapperFunc().then(result => {
    // got final result
}).catch(err => {
    // got error
});

How do I split a string into an array of characters?

You can split on an empty string:

var chars = "overpopulation".split('');

If you just want to access a string in an array-like fashion, you can do that without split:

var s = "overpopulation";
for (var i = 0; i < s.length; i++) {
    console.log(s.charAt(i));
}

You can also access each character with its index using normal array syntax. Note, however, that strings are immutable, which means you can't set the value of a character using this method, and that it isn't supported by IE7 (if that still matters to you).

var s = "overpopulation";

console.log(s[3]); // logs 'r'

Adjusting HttpWebRequest Connection Timeout in C#

Something I found later which helped, is the .ReadWriteTimeout property. This, in addition to the .Timeout property seemed to finally cut down on the time threads would spend trying to download from a problematic server. The default time for .ReadWriteTimeout is 5 minutes, which for my application was far too long.

So, it seems to me:

.Timeout = time spent trying to establish a connection (not including lookup time) .ReadWriteTimeout = time spent trying to read or write data after connection established

More info: HttpWebRequest.ReadWriteTimeout Property

Edit:

Per @KyleM's comment, the Timeout property is for the entire connection attempt, and reading up on it at MSDN shows:

Timeout is the number of milliseconds that a subsequent synchronous request made with the GetResponse method waits for a response, and the GetRequestStream method waits for a stream. The Timeout applies to the entire request and response, not individually to the GetRequestStream and GetResponse method calls. If the resource is not returned within the time-out period, the request throws a WebException with the Status property set to WebExceptionStatus.Timeout.

(Emphasis mine.)